Вы находитесь на странице: 1из 172
Mathematics Higher Level for the IB Diploma Solutions Manual Paul Fannon, Vesna Kadelburg, Ben Woolley and Stephen Ward ep CAMBRIDGE AB UNIVERSITY PRESS CAMBRIDGE UNIVERSITY PRESS ‘University Peating House, Cambridge cx? 81s, United Kingdom “Cambridge University Press is pat of the University of Cambridge, ‘Mfuthers the University’s mission by disseminating knowledge inthe pursuit of ‘cdseation, learing and research a the highest international levels of excellence, srmwcambridgeorg Taformation on this til: education cambridge ong (© Cambridge University Press 2016 ‘This publication isin copyright Subject to statutory exception and fo the provisions of relevant cote licensing agreement, ho reproduction of any part may take place without the writen permission of Cambridge University Press, First published 2016 Printed in Indla by Multivista Global Pvt Lad ‘A catalogue recon for this publication is available rom the British Library ts 978-1-107-57937-8 Paperback. Cover image: Craig JewellShutterstock ‘Cambridge Univesiy Press has no responsibilty for the persistence or accuracy STURT for external or third-party internet websites reerted ton this publication, fand doesnot guarantee that any content on such websites is or will remain, {eaurate or appropriate. Information reading prices, travel tintetables, and other factual information given inthis works correct atthe time of first printing but Cambridge University Press does not guarantee the accuracy of sch information thereafter Tei legal to reproduce any pat of this workin material frm (induding photocopying and electronic storage) except under the following circumstances: () where you ate abiding bya licence granted to your school or institation by the Copyright Licensing Agency (i) where no such Fence ens o where you wish to exceed the terms Fa licence, ‘nd you have gained the written permission of Cambridge University Press, ii) where you are allowed to epoduce without permission under the provisions ‘Of Chapter 3 of the Copyright, Designs and Patents Act 1988, which covers, for txample, the reproduction of short passages within certain types of educational Snthology and reproduction forthe purpases of setting examination questions. Contents Chapter 1 Chapter 2 Chapter 3 Chapter 4 Chapter 5 Chapter 6 Chapter 7 Chapter 8 Chapter 9 Chapter 10 Chapter 11 Chapter 12 Chapter 13 Chapter 14 Chapter 15 Chapter 16 ‘Chapter 17 Chapter 18 Chapter 19 Chapter 20 Chapter 21 Chapter 22 Chapter 23 Chapter 24 Chapter 25 Chapter 26 Introduction Counting principles Exponents and logarithms Polynomials Algebraic structures ‘The theory of functions ‘Transformations of graphs Sequences and series Binomial expansion Circular measure and trigonometric functions ‘Trigonometric equations and identities Geometry of triangles and circles Further trigonometry Vectors Lines and planes in space ‘Complex numbers Basic differentiation and its applications Basic integration and its applications Further differentiation methods Further integration methods Further applications of calculus Summarising data Probability Discrete probability distributions Continuous distributions Mathematical induction Questions crossing chapters “This book contains worked solutions for all the exam-style questions that, in the Higher Level coursebook, are colour-coded green, blue, red or gold. As a reminder: Green questions should be accessible to students on the way to achieving a grade 3 or 4. Blue questions are aimed at students hoping to atain a grade 5 or 6, Red questions are the most dificult, and success with a good number of these would suggest a student is on ‘course for a grade’. Gold questions are of a type not typically seen in the exam but are designed to provoke thinking and discussion in order to develop a better understanding of the topic. Of course, these are just guidelines. If you are aiming for a grade 6, do not be surprised if occasionally you find a green question you cannot do; people are rarety equally good at all areas of the syllabus. Similarly, even if you are able to do all the red questions, that does not guarantee you will get a grade 7 ~ afer all, in the examination you will have to deal with time pressure and exam stress! Its also worth remembering, that these questions are graded according to our experience ofthe final examination. When you firs start the course, you may well find the questions harder than you would do by the end of the course, so try not to get discouraged! “The solutions are generally writen in the format ofa ‘model answer’ that students should aim to produce in the exam, but sometimes extra lines of working are included (which wouldn't be absolutely necessary in the ‘exam) in order to make the solution easier to follow and to aid understanding, In many cases the approach shown is not the only option, and neither do we claim itto always be categorically the best or the approach that we would advise every student to take; this is clearly subjective, and different students will have different strengths and therefore preferences. Alternative methods are sometimes given, cither in the form of a full worked solution or by way of a comment after the given worked solution. Where the question has a calculator symbol next to it (indicating that itis for the calculator paper), the approach taken is intentionally designed to utilise the calculator and thereby to minimise work. Students should make sure they are familiar with all the calculator techniques used and, if not, the calculator skills. sheets on the CD-ROM accompanying the coursebook should be consulted. When there is no symbol (indicating that the question could appear on either the calculator or the non- calculator paper), the solution given usually assumes that a calculator is not available. Students should make sure they can cope in this situation but also that they can adapt and use features ofthe calculator to speed up the process. Perhaps the most common example of this is to use the calculator’s equation solver rather than solving simultaneous or quadratic equations manually. ‘We strongly advise that these solutions be consulted only after having spent a good deal of time and effort "working on a question, The process of thinking about the problems encountered, even ifa ful solution cannot ultimately be found, is really important in developing the skills and knowledge needed to succeed in the future. ‘We hope you find these solutions useful and that, when used wisely in conjunction with the coursebook, they lead to success in the IB exam. Stephen Ward, Paul Fannon, Vesna Kadelburg, Ben Woolley Ba y 1 Counting principles Exercise 12x4=48 a 5X6x3=90 b 6x(5+3)=48 © 5X6+5X3+6x3=63 © 5x3=15 B® 58x68x61x65=15637960 b 15637960x2= 10104 528 26° x9* = 115316136 ‘There are 4 options for the journey to the middle, then two possible directions (left or right), and then a single choice of path upwards, So the total number of possible routes is 4x2x1=8 a 15x4x12=720 There are two cases: «shirt is not pink = no constraints on tie or waistcoat: 15x4x9=540 + shirt is pink = tie not red, waistcoat not red: 12X3x3=108 Total number of possible outtits is 540+108 = 648 a 5x4x3=60 b S*=125 a 3*=81 b S125 Exercise @ @ a 7!=5040 b Number of arrangements with largest book at one end: 2x6!=1440 So number of arrangements with largest not at either end: 5040-1440 = 3600 a 5!=120 b Require the final digit to be 5, so there 24 such numbers. (© 2 171=35568 742809600 b 16!= 20922789 888000 @ s1x4!1=2880 @ « 6t=720 bb Require the first digit tobe 1 or 2, so there are 2!x5!=240 such numbers. © 30!=2.65x10" (sk) Because there are an odd number of red toys, the one in the middle must be reds then, on the left arrange two red and two blue, to be matched on the right. The following colour patterns are possible on the left: RRBB RBRB RBBR BRRB BRBR BBRR ‘There are 21x21 available. colour patterns Sounding inci a 7 Within each pattern, there are 5!x4! arrangements of the individual toys. So total number of possible arrangements is 5!x4!x6=17280 Exercise a Geant =20 1 —n-20=0 (n-5)(n+4)=0 =3n=5 (asn isa positive integer) (Dnt -(n—1)!=16(n-2)! Dividing through by (n—2)! n(n—1)-(n—1)=16 n—2n-15=0 (n-5)(n+3)=0 =on=5 (forneN) Exercise © Choose 9 from 15: 15) _ 5095 9 |* a Choose 3 from 7: Cee b Choose 2 from 6: (> Topic 1D Counting selections 39° [> |=15380937 7 Choose 3 boys from 16 and 2 girls from 12: 16) 12) _ 560566436960 93 J 7 350x66= 36 © Choose 1 goalkeeper from 3, 4 defenders from 8, 4 midfielders from 6, and 2 forwards from 5: 3) (8) (6) (5 xt let x ACCC =3x70x15x10= 31500 ©) Choose 12 from 140, then 10 from 128, then 10 from the remaining 118: (} (‘) (‘) x) |x 12)*(10) "(10 =1.61x10° (3SF) © 2 Choose 3 boys from 14 and 2 girls from 16: i x} bd |= 364 120=43 680 3 [2 Pepans b Choose 3 boys from 14, and for the girls choose 2 from 15 (the 2 places not taken by Roberta or Priya) and choose 1 from 2 (Roberta or Priya): 14), (19) c2=364x105x2=76440 3 lo Pee x 2= (BY 2 Exactly 6¢ is spent if you choose 3 of the 2¢ sweets from 7: > b Exactly 7¢ is spent if you choose 1 of the 2¢ sweets from 7 and 1 of the 5¢ sweets from 5: (s)- D «Exactly 10¢ is spent if you choose 5 of the 2¢ sweets from 7 or 2 of the 5¢ sweets from 5: ceo [At most 5¢ is spent if you choose 1 or 2 of the 2¢ sweets from 7 or 1 of the 5¢ sweets from 5: (Hamm 9 Choose 4 from 9: ()- 126 Exclude the possibilities where all questions are from the same section. All questions from A: choose 4 from 5: All questions from B: choose 4 from 4: () z- number of ways of choosing at least one from each section is 126-5~1=120 (© To deliberately double-count: each point connects to 14 other points s2n= 15x14 5X7 =105 lines COMMENT Its assumed that vertices of the triangles/ quadrilaterals can only be at the original fen points and not at any intersections created by lines joining these ten points 4 Each triangle is defined by three points. Number of different sets of 3 points = 0 = 120 3 | Each quadrilateral is defined by four points. Number of different sets of 4 points = ‘hm (} With » people, each shakes hands with n—others. This double-counts the total number of handshakes. ennt nt’ —n—552=0 (n=24)(n423)=0 =na24 (] Once the rows are determined, there is only one arrangement for each row. Choose 15 from 45 for the first row and 15 from 30 for the second row: 45) (30 u : x| 7) |=3.45x10" x1.55%10" 15) "las =5.35%10” (3SF) Exercise Numbers divisible by 5 range from 21 x 5 to 160 x 5: 140 multiples of 5 So there are 700—140= 560 numbers not divisible by 5. LOMMENT, Alternatively, 4 out of every 5 of the numbers between 101 and 800 are not divisible by 5, so there are $x700- 560 of them _ ©) There are 6 different letters. ‘Total number of permutations: 6!=720 Number of permutations beginning with 51=120 total number not beginning with = 720-120 = 600 ©) @ Choose 3 from 10: 10) 120 3 | 'b Choose 3 from 22, then exclude the choices of all chocolates: 3 from 12 22) (12 |-| , |=1540-220= 1320 3) \3 ‘There are 7 different letters. ‘9 Total permutations: 7!=5040 Permutations beginning with ‘KT’ 51=120 «total not beginning with ‘KI’ 5040—120= 4920 Permutations beginning with ‘KT or ‘TK’: 2x5!= 240 «+. total not beginning ‘KI’ or ‘IK’: 5040-240 = 4800 ‘Total committee possibilities: 2 20349 ae ‘Total all-male committees possible: 12 C= 5 «=. total committees that are not all male: 20349-792= 19557 © Total possible selections of 7 tiles: ©) _ 657800 7 | ‘Total possible selections with no vowels: 21) 116280 aE re rt ‘Total possible selections with one vowel: 5=271320 2h) 5 6 +s total possible selections with at least two vowels: 657800—116280— 271 320= 270200 22 © ‘otal possibilities: | |=74613 ‘Total possibilities with no women: 10 ( = 210 6 ‘Total possibilitiés with no men: 12 ( ) 924 6 ‘Total possibilities with exactly one man: 12) 10) x |=7920 5 1 +. total possibilities with at least 2 men and 1 woman: 74613—210-924~7920 = 65 559 ©} Among the integers 1-19, there are 10 odd numbers and 9 even numbers. a No even numbers =9 choose 7 from 10: 10 = 120 (") One even number => choose 6 from 10 4.1 from 9:( 1° )x{?|=1990 and 1 from 9:} 5 |x| ‘Two even numbers => Choose 5 from 10 and 2 from 9: (Sha) . choices with at most two even numbers: 120+1890+9072=11082 b Total possibilities => choose 7 from 19: ) 50388 7} *. choices with at least two even numbers: 50388 — 120-1890 = 48378 7 D ©) Total permutations: ‘Total beginning with ‘D’ and ending with 24 ~. total not beginning with ‘D’ or not ending with ‘L: 720-24 = 696 Exercise @ 2p =77519922.480 (© *P,= 255024 @’R=504 Q'n=26 © *2,x"P, =3276000 (0) there are 7 different letters, consisting of 3 vowels and 4 consonants. ‘Total number of permutations: ’P, = 210 Total number of permutations with no vowels: *P, =24 «total permutations with at least one vowel: 210-24=186 © Other than James, choose 2 runners 7 from 7: | |= from 7: ()) a Arrange 3 nanners in medal positions: 31=6 In two-thirds ofthese arrangements, James isin first or second. Total valid arrangements: 21x6% 3 =84 © Case 1: Rajid not selected. Choose 3 students from 17 and permute Case 2: Rajid is selected. Choose 2 students from 17, select one post from two for Raji, then permute the other two candidates in the two 17) rnin ("st ‘Total valid arrangements: 4080-+544 = 4624 mp6, 6 (6-n)! Since P, isa real value, n <6 => 2n(2n-1)(2n-2 Since *P, is a real value, n> 2 ‘Trying all possible values: n=2: ‘P,=4#°P,=30 n=3: P,=120="P, SP, =336#360= °P, "p =720="R n=6: "P,=1320#720=°R, So n=30or 5 Exercise ‘Treating Joshua and Jolene as one unit, permute 13 units, then internally permute Joshua and Jolene: 13! x2=12454041600 ©) Arrange the three blocks in 3!=6 ways. Internally permute the members of each class in 61x41 4! ways. ‘Total arrangements: 6x6! x4! x 4!= 2488320 ©) Treating the physics books as one unit, permute 7 units; then internally permute the 3 physics books: 71x31=30240 1 Counting principles _ ©) The 13 Grays and Greens can be arranged in 13!= 6227020800 ways. ‘There are 14 spaces in the line-up. (including the ends), and one Brown must stand in each of 4 of these spaces. 14 “There are ( *}pr90 possible space selections. ‘Then permute the Browns: ‘Total possible arrangements: 131x1001x4!=1.50x10" (3SF) ‘Sp = 32432400 b There are 9 possible spaces for the leftmost friend to sit. Total arrangements: 9X7!=45360 ‘Treating the men as one unit, permute 6 units, then internally permute the 4 men: 61x4!=17280 ‘Treating all the men as one unit and all the women as one unit, permute the 2 units, then internally permute the 4 men and internally permute the 5 women: 21x41x51=5760 Permute the 5 women: 5! Into 4 of the 6 spaces, insert one man, and then permute the men: €P, =360 ‘Total possible arrangements: 120x360 = 43200 Require a WMWMWMWMW arrangement, Permute the 5 women and the 4 men: 51x41 = 2880 mh P Mixed examination practice 1 Short questions Choose 3 from 7 and permute them: 7 ( este? =210 3 BD Permute 5 units: 5!=120 Permute 3 and permute 7: 3!x7!= 30240 g=729 ‘Total possible choices without restriction ~ does ont) Choices which contain the two oldest ~ choose 2 from the remaining 6: = .+ choices not containing both of the oldest: 70-15=55. GB s!=30(n-1)! (n-+1)(n)(n-1)!=30(n-1)! n(n+1)=30 nm +n-30=0 (n-5)(n+6)=0 ‘con=5 (eject the negative solution n =~6) (©) There are 8 different letters, consisting of 4.vowels and 4 consonants. Choose 1 of the 4 consonants to be the first letter, 1 of the other 3 consonants to be the last letter, and permute the remaining 6 letters for the centre: CH) {© Choose 1 from 5 to be the driver and permute the remaining 7: n=n-210=0 (7 }ert=2sam (n—15)(n+14)=0 nels (reject the negative solution n =-14) COMMENT Notice that the exact arrangement of people in each row of the other seats is 15 ne 5 from 15:| . |=3003 irrelevant, since each seat is uniquely choose ( 3) idenified. The answer would be the Choices which are all girls: choose 5 same for a van with seats in a 2-2-2-2 3 ‘arrangement, for example, ome) = 105 {© toa possible choices without restriction: «. choices which contain at least one boy 3003-56 2947 (©) The drivers can be arranged in 2 ways; then choose 3 from 8 to goin the car: ‘Total permutations without restriction: 8 6!=720 x) = 112 Permutations containing BE or EB: permute 5 units and then internally permute B and E:5!X2!=240 ‘COMMENT ~. permutations without Band E next to The specific people fo go in the van need each other: 720-240= 480 not be considered, since after choosing those to go in the car, the rest will go in the von. Because (3)-(3} it makes no difference which vehicle is considered to Choices which contain the two youngest: have the first pick of passengers when choose 3 from the remaining 10: making the calculation. 10 ao aI + choices not containing both of the youngest: 792—120= 672 ©) © Choose | of 2 places for Anya, then permute the other 4: 2x4!=48 1) Total possible choices without restriction: 12 choose 5 from 12: (: =m Long questions © Choose and permute 3 letters from 26: Bx ( *) =p = 15600 Total possible permutations: 5!=120 3) Permutations with Anya not at an end: Choose 5 digits with repeats allowed: 8-22 9° =59049 With Anya at the left, permute the <1 total possible registration numbers: other 4: 41= 24 1560059 049=921164 400 With Elena on the right, permute the other 4: 4!=24 &_|)Counting principles other 3: 3!=6 To find the total number of possible permutations, remove the double- counted cases: 24+24-6= 42 ©) « Unrestricted ways of sharing: 2° = 32 Cases with all sweets to one person: 2 Cases with at least one sweet to each person: 32—2=30 b 2% =32 ¢ Choose 1 sweet from 5 to be (> split“ }= Choose 2 sweets from 4 for the first “es person:| 5 |= (Then the second person gets the rest of the sweets.) Total possible choices: 5x6=30 © « Choose 8 seats from 14 and permute 14 the 8 people: { 5 starz 08050 b> Consider the people of the same family asa single unit. Choose 6 seats from 12 and permute the 6 units, then internally permute the 3 people of the same family: 12 og XOLx31= 3991 680 © The person with the cough can sit at an end seat with one empty seat next to it or in a non-end seat with an empty seat on either side. Case_1: Choose an end for the cougher, choose 7 seats from the remaining 12 and permute the 7 people => Oo: a Case 2: Choose 8 seats from 12 and permate the 8 => 12 jg Pest= 19958400 ‘Alternatively, choose 1 seat from 12 non-end seats for the cougher, choose 7 seats from the remaining 11 and permute the 7 people => BR) (1), x i)*\7 Total possibilities: 7983360+19958400= 27941760 9958400 Ifthe four slots are labelled 1, 2,3 and 4, choose two of these to be the positions for the ‘R's (then the other ‘wo will automatically be the positions for the ‘D3); this is effectively choosing 4 2 slots from 4, which gives| (} By the same logic as in (a), m ‘Rs and n 2r “p's can be arranged in ( ”) ways. ® ‘To get from top left to bottom right in a4 x4 grid, the miner must make 3 moves to the right (R) and 3 moves down (D), ie. in some ordering of 3 ‘Rs and 3 ‘D5. By (b) there are(S)-20 a By the same argument as in (a), (n—1) ‘Rs and (m=1) ‘D's can be arranged in (7) ways. n-1 © «Choose 6 people rom 12 for one tans (2) ws However, this will double-count, because choosing 6 people for one team is equivalent to choosing the other 6 people for the other team, since the two teams of equal size are not specified as A and B. 2 ‘The total number of possible teams is therefore x j \ Choose 1 from 12, 2 from 12, 3 from 12, 4 from 12, 5 from 12 or the previous answer: 1) (12) (22) (12), (12) 1/12), ra sass Jal o> Choose 4 from 12, then 4 from 8. But this will over-count by the number of rearrangements of the 3 groups, ie. 31, s0 P)(8 a a1e5775 a)*a)* 31 choose tom i ( =a Choose 3 from m: ()-s0 n(n—1)(n-2) 3! 1 —3n? +2n—9240=0 From GDC: n= 22 = 1540 Choose 3 from m({)=t00 n(n-1)(n—-2) pe n(n? —3n-+2)=600n 1? —3n-598=0 (reject n=0) (n—26)(n+23)=0 26 (reject negative solution n=-23) < Searing cil Exponents and logarithms Exercise @Y ©) vinputs are sorted in kxn* microseconds. 1 million=10? inputs are sorted in 0.5 seconds=0.5 10° microseconds. c kx(10°)" =0.5x10° x10" =0.5x10° k=05x10" =5x10% -xy?, and when x=2y, V= 128. 2y)x y? = 128 ya64 ya Hence x=2x4=8cm. (D « Substituting A=81 and V=243: veka 243 = kx(81) 2as-kx(s1')) 243 =kxo? 243=729k ket 3 (D 2° =(27)? =128* >125* =(3")° COMMENT This question lends itself fo a comparison where either bases (not possible in this case) or indices can be made to match, Allornatively, some standard ‘approximations can be used which, iF known, allow o different approach. You may find it useful fo know that 2=1024 = 10° and 51°=9765 625 ~ 107, Then the following working gives o proof: 210 =1024 >10° 285 = (200) (108) «1010 5° =9.77 «10% <107 238190 (510) <(107)* = 101% 2 2350 5 5150 Multiple solutions for x means that this is an undefined value, so 2a ~ 3=0=log,b. ©) _4209:00 on Tuesday, Substituting: y=kxL 10=kxL k=10 ‘At 09:00 on Friday, ¢=3. + y=10X1.P=13.31 m? T=A4Bx2* a Background temperature is 25°C, so as xo, T 25. Since2* 0, TA. Hence A=25. ‘Temperature on surface of light bulb is 125°C, so when x=0, T=125. Substituting: 125=25+Bx2° => B=100 ‘Air temperature 3mm from surface of light bulb is 75°C, so when x=3, T=75, Substituting: At 2cm from the surface of the bulb, x=20 (mm), so 2» T=25+100x2 > =26.0°C ~x 3 Figure 2B.3 Graph of T= 25+100x2 2 A 5 m Figure 2B.4 Graph of h= 2-0.2x1,607" pe? b- When there is no fruit, m=0, so Figure 28.5 Graph of y= 1+ 16 h=2-0.2X1.6°=1.8m_ ¢ When m=7.5, COMMENT h=2-0.21,6°75=1.60m (35F) Some justification of how this graph could be constructed is given here, but you The model was derived from data could just draw iton a GDC. which, from (c), gave a height of 1.6m above the ground at the harvest-time fruit load of 7.5kg, Extrapolating so far beyond the model to reach h=0 is unreliable and likely to be unrealistic; for example, the branch might simply break before being bent far enough to touch the ground, yeltle™ =1+ 1 Asx es, 0and soy 1. o £0,866 (3SF) ‘The maximum value for y must occur at the minimum value of x? ~1, which is when x=0. (3) Temperature T of the soup decreases 1 exponentially with time towards 20 When x=0, y=1+——7=17, 0 the (room temperature): 16" : ‘maximum point is (0,17) TH 204 Arm When the soup is served, ¢=0 and T'=55, 2.55=204+Axm! A=35 12 bye 28 Epona fotos Every 5 minutes the term Ax" must decrease by 30%, so m=0.7 and 1 kes “T= 2043507" ‘When t=7, T=20+35x0.7"4 =41.2°C (3SF) b Aste, 3°" Oand so V 40(1 - 0)=40 ms* Exercise (©) This is a very close approximation toe (with less than a 7.5% 10-7% error). COMMENT It might ot first seem that this is Far too unlikely fo be @ mere coincidence and that there must be some underlying relationship, but ths isin foct not the ‘case, The socalled ‘Strong Low of Small Numbers’ gives some insight into the surprisingly regular occurrence of this type of coincide In(@x-1)=2 3x-1=8 (1) flog, x+log, y=6 ... log, x-log, y=2 ...(2) (1) +): 2log,x=8 log,x=4 x=3'=81 Substituting into (1): Atlog, y=6 log, y=2 ye ie. x=81, y=25 (©) 30-+logx)=6+logx 3+3logx =6+logx 2logx=3 3 logx == logx=5 x=10! =10V10 =31.6(3SF) 2 ponents ond earitins 13 ) x24} 21170382) (B Let rbe the Richter-scale value and s the strength of an earthquake. Since an increase of one unit in r ‘corresponds to an increase by a factor of 10 ins, s=Cx 10 for some constant C. Let r be the strength of an earthquake of Richter level 5.2: t=Cx10...(0) For an earthquake twice as strong: 2t=Cx10" ...(2) 2t_ Cx10" (2)+ (0): == OO T= Cx 2=10* 1-5.2=log2 r=log2+5.2=5.50 ‘An earthquake twice as strong as a level-5.2 quake would measure 5.5 on the Richter scale. COMMENT The constant C is needed here as the precise relationship between rand s is not given, but itis not necessary fo find the value of C to answer this particular question. © @ ins0=1n2+1n25 =In2+In5* =In2+2In3, =at2b 4 b tno16=In( *) =In2?-Ins* =2in2-2In5 =2a-2b log, x=log,2 log, x= 12822 Ba Tog x (log, x)° =1 log,x=41 1 =2'=2 or = * 2 =(ab)” xIna=xylnab x(Ina~ylnab)=0 Ina ___Ina 9880 8 eT inatinb bY =(ab)”” ylnb=xylnab y(Inb—xInab)=0 = y=0 of x= th __Inb * “Ina Inatinb x=05a*=1,s0b’=1 and hence y=0 (since a,b>1) _Inb+Ina vecither x= y=0 or xt y= 123 89 logs + logs + log; +...tlog.s Hog faa 1 © tog, b=Iog, a Using change of base: logh _ loge Joga logh (logb)’=(loga)” logh= +loga logb= log(a"") beat! = log) Reject b=a sb=a" hy Exercise ‘The domains are different; y=2log x has domain x > 0 whereas y=log( x?) thas domain x# 0 and is equivalent to y= 2log|s}. COMMENT The rule of logarithms that logx? = plogx only applies to positive x.’ Exercise B Nz=100e'™ a When t=0, N=100¢"=100 When t=6, N=100e'" x*=48300 (3SF) 1,03t=1n10 t=—Linto 1,03 = 2.24 hours (3SF) The population will reach 1000 cells after approximately 2 hours and ©) Let Nbe the number of people who know the rumour f minutes after 9 a.m. Then N can be modelled by the exponential function N= Ae” 2 Exponents and lagarithns ‘At 9 a.m. 18 people know the rumour, so when f=0, N=18: 18= Ae? = A=18 At 10 a.m. 42 people know the rumour, so when (=60, N=42: ‘Therefore N= 18e""", a At 10:30 a.m, 1=90, oN a1 = 64.2 So 64 people know the rumour at 10:30 a.m, If 1200 people know the rumour, then 1200=18¢0#" So after 297.4 minutes (4.96 hours), Le. at 13:58, the whole school population will know the rumour, P=32(e8#02"_ 1) a 2 minutes=120 seconds P(120)=4.96 units per second (from GDC) = 18 Topic 2G Sobing exponential equations. 0012 32 = 8327.6 So the experiment can be run for £8328 seconds, equivalent to 2 hours, 18 minutes and 48 seconds. parades n+ ) Ww ke t Figure 2G.5 Graph of W = ke °°! 25% of the inital mass means W =. kygtte = 138.6 seconds ‘The block will be at 25% of its original weight after 2.31 minutes. COMMENT 138.6 seconds is 138.6 + 60= 2.31 minutes. COMMENT The answer in the back of the coursebook, A ) is an equivalent, though not was fully simplified, form. log,3 x=10+log,3 ©) Let K CO) be the temperature after tseconds; then K-22=Ae* K(0)=98 > A=76 K(120)=94 => 72=76¢"%* ou) 0.000451 (3SF) 120 (76. als) 0.000451 \ 76. =678 (3SE) It takes 678 seconds, equivalent to 11 minutes and 18 seconds, for the tea to cool to 78°C, ‘y=3" is always increasing and y=3 ~ x is always decreasing, so their two ‘graphs can intersect at most once. Since 3” <3 ~O and 3! > 3 ~ 1, there must be an intersection in [0, 1]. From GDC, the intersection occurs at x=0.742(3SF) Mixed examination practice 2 Short questions OD 0e, (Va? +49)=2 x? +49 =25 2? +49=625, x? =576 x=i24 Do tog 22 toga? +1057 -loge z 4 =logx? +log y? —logz = 2logs + Hlogy-loge =2arb—¢ 2 2.Exponents ond logarithms b logV0-ix =log(0.1x)* = Flogo..x = (log0.1+ gx) = (1+ ogs) © inx+iny?=8 Ins? +Iny=6 Inx+2Iny=8 2inx+Iny=6 2x(2)-()s 3inx=4 4 Inx=— 3 x=e! =3.79 (3SF) Substituting in (2): © y=inx-in(x+2)+In(4-x?) =u 2)) x2 ~o( 200+) x42 _2V4—4e” “2 _242VI-e” ar) itvi-e” COMMENT To approach this type of question, try to rewrite itso that all terms involving the unknown x have either @ common base ‘or a common exponent. Betyg a 36 2 x(2) x3? x(3)f = 367 x36" 1 x36" Lxsex tng 4 27 (2) 82 36 223 In3 Ind 36 {(} Changing log, and log, into in: Jog, b= = 2log,b=¢ 2(Inb)’ -(Ina)* =Inalnb (ina) +Inb (Ina)-2(Inb)* =0 ‘Treating this as a quadratic in Inaand factorising: (Ina—Inb)(Ina+2Inb)=0 z-Ina=Inb or Ina=—2inb alah ghd? = yp ie. a=b or a=e* But we are given that a < b, soa # band hence a=b". Slog, x=25log, 5 Using change-of-base formula: Slogx _ 2Slogs “Togs log (logs) = (logs) Long questions BD a Asta, e% 40 and so V>42 When t=0, V=42(1-e°)=0 ¥ y=42 t Figure 2ML.1 Groph of V = 42(1-6°®2") b> When t=0, V=42(I-e")=0ms* © As tes, e% 0, “Vo42ms* COMMENT The graph in (a) can be drawn immediately with a GDC; the answers to [band can hen simply be deducod from the graph without frst caleulating V when 1=0 ond the limiting value of V as to, 2.Exponents and logarithms 37000 tigers in 1970, ie. when n=0, T=37000. 1 37000=ka® k=37000 22.000 tigers in 1980, i.e. when n=10, T=22000, ©. 22000 = 370002" a2 37 2a F ate 949 (3SF) Hence T=37000%0.949". In 2020, n=50: T=37000x0.949"=2750 When T=1000: 1000 = 37000%0.949" ogast = + so tigers will reach ‘near extinction’ in 2039, Under the initial model, in 2000 (n=30) the number of tigers is 778 kb", has T=7778 when m=0, s0 k=7778. Under this model (T=7778b"), there are 10000 tigers in 2010, ie, when m=10, T=10000. fixed examination prociaal2 ‘Therefore the new model is T=7778x1.025m. ‘The growth factor each year is 1.025, equivalent to a 2.5% growth rate. Iny=2inx-+In3=In(3x") = y=3x? (Note that x > 0 for the original relationship to hold.) Iny=4Inx+6 =In(x*)+In(e*) =In(x‘e*) = yae’xt Iny-2=3(x-1) Iny=3x-1 syse™ e =4x7 => y=In(4x*)=In4+2Inx So the graph of y against In xis a straight line with gradient 2. 3 Polynomials Exercise G Factorised form is more useful for finding roots; expanded form is more useful for evaluating the y-intercept and for comparing, adding or subtracting polynomials. Yes; the term in x" is unaffected by adding a lower-order polynomial. No; ifthe lead coefficients have a zero sum, then the sum of the polynomials will not have a term in x", so the resultant will be of lower order. For example, the sum of the third- order polynomials f (x)= x’ ~2x and g(x)=3+x-a7is f(x)+g(x)=3-x a polynomial of order 1. Exercise MM ENT Icon be helpful in questions to label « function as F(x) org(x),s0 that evaluating «at particular values of x can be clearly described CD F(x)=6x' 40x? +bx+8 By the factor theorem: f(-2)=0=—48+4a-2b+8 4a-2b=40 =b=2a-20 ... (1) By the remainder theorem: f(l)=-3=6+a+b+8 =a+b=-17 ... (2) Substituting (1) into (2): 3a-20=-17 BD fl(x)s0 48x? +ax+b By the factor theorem: =0=84324+2a+b oo) By the remainder theorem: (f(3)=15=27+7243a+b @) Substituting (1) into (2): By the factor theorem: Sf (k)=0=k +k? -8k 2k -8I K(k-4)= k=0 or k=4 MA) R B f(a -(k+1)x-3 © flx)=x tax? 49x40 By the factor theorem: a By the factor theorem: f (kl k-1) ~(k+1)(k=1)-3 f(11)=0=1331+121a+99+b K-2k+1- +1-3=0 = b=-1430-121a -2k-1=0 By the remainder theorem: 1 f (-2)=-52=-8+4a-18+b 2 = 4a+b=-26 © fle)= 2 ax? -bx +168 <-4a~1430-121a=—26 -117a=1404 a. By the factor theorem: con ben f(7)=0=343—49a—7b+168 = b=-7at73 By the remainder theorem, the = remainder when divided by (x-2) is S2): f(3)=0=27-9a—3b+168 = 9a+3b=195 f(2)=8+4a+18+b £9a43(-7a+73)=195 = 48+18+22=0 -12a=-24 ‘That is, (x2) is a factor of f(x). >a=2, © fle)ax tax? +3x+d bf (x)=(x-3)(x-7)(x-k) By the remainder theorem, Expanding: f(-)=6=-1+a-3+b f (x)= x? (104k) x? +(214+10k)x—21k =atb=10 ‘The remainder when divided by (x1) is FQ) f(l)=1+a+34+b=143+10=14 Comparing coefficients: x l0+k=a=2>k=-8 x! :21+10k=-b=~59, consistent with k=-8 o x°:-21k=168, consistent with k=—8 So the remaining factor is (x+8). SAMHERY There are two sensible approaches here. You may recognise that the given COMMENT quadratic factorises readily into eh an of . . (x-2)(x=3), soit would be possible to Only one of he cooficien! comparisons ppl the factor theorem tothe cubic and However iis good practice fo quickly solve f(2)=f(3)=0 t0 find a.and b. very whothereriten down ion exam Alternatively, propose a final factor vita oath thatthe cr comparisons [2x-K), chosen fo ensure that the lead ‘are valid, 1o check for errors in working. oA er anit, methods are shown below. 22 Topic 9B Remoinder and factor theorems f(x)= 2x? 15x? tax tb Method 1: 42 -5x-4+6=(x—3)(x=2), 80 both (x~2) and (x3) ate factors of f(x). By the factor theorem: f(2)=0=16~60+2a+b = b=44-2a Also, f(3)=0=54-135+3a+b =)3a+b=81 = 2x? +(-k-10)x* +(5k-+12)x—6k Comparing coefficients: xt: a=5k+12=37 x! b=-6k=-30 COMMENT Although the two methods are of similar difficulty, the first requires that you spot the factors of the quadratic quickly, which is not needed for the second method. The second method also produces the final factor, which may be useful in a multi-part question. Exercise G&@ ©) Repeated root at x= 2, single root at x =3; y-intercept at 24. Negative cubic shape. y Figure 3C.7 Groph of y = 2{x+2)?(3—x) ©) o Repeated root at x=3 = factor of (x3) Single root at x =-2 => factor of (x+2) =k(x-3) (x+2) = k(x? 4x7 -3x+18) y(0)=36=18k => k=2 "y= 2x" 8x? 6x +36 ie, p=2, q=-8, r=-6, 5=36 b_ Repeated root at x=0 => factor of x* Single root at x=3 => factor of (x-3) y= ke (x-3) y(2)=4=—4k = k=-1 a 7, 1 YD aah a(utat)(e—at) =(¥+9°)(x+4)(2-4) b Roots atx=tg only; +y-intercept at—q* Positive quartic shape. Even function (reflective symmetry about the y-axis). y Figure 3C.9 Graph of y = x4 —4' (D « Repeated root at x= p, single root at x=q: y-intercept at —p7q. five cubic shape. y Figure 3€.10 Graph of y =(x-pl’Ix-a) b From the graph, there will be only one solution for y=k when k>0. Pr Exercise © Equal roots when discriminant is zero: A=b?—4ac=0 (4 -4xmx2m=0 16—8m? =0 mi m=tv2 (© Tangent to the x-axis implies equal roots, so discriminant is zero: A=b?—4ac=0 (2k+1)° -4x(-3)x(—4k)=0 4K + 4k+1-48k=0 4k? 44k+1=0 M48 VAP 4x41 2x4 44+ 1920 k @ Fora quadratic to be non-negative (2 0) for all x, it must have at most one root, so ASO and a>0, bP -4acs0 (-3)'-4x2x(2c-1)s0 9-16e+850 Ww ca 16 COMMENT Note that A<0 is not sufficient in general for a quadratic to be non-negative. The condition a>0 is also necessary to ensure that the quadratic has a positive shape (opening upward) rather than a negative shape (opening downward), so that the curve remains above the xaxis and never yes below it, as would be the case if ‘a<0. In this question a was given os positive (2), so we did not need to use this condition at all. (© No real solutions when A<0: (-2k)'-4x1x6k <0 4k? 24k <0 k(k-6)<0 O bis positive (as a is negative) TABLE 3MS.2 Positive ©) Repeated root at x=-3 = factor of (x43) Single root at x =1 = factor of (x—1) Single root at x= 3 = factor of (x—3) ry = k( x43) (x-1)(x-3) = k(x*+2x?- 12x? -18x+27) y(0)=27>k=1 yax* 2x? 12x? ~ 18x +27 =2, c=-12, d=-18, e=27 26 Mixed examination practice 3 f(e)=(ax+b) By the remainder theorem: flx)= 4x7 +x46 a f(2)=8-16+2+6=0, so by the factor theorem, (x—2) isa factor of f(x). b f(x)=(x-2)(x?+ax+b) =x? +(a—2)x? +(b—2a)x—2b Comparing coefficients: xt: a-2=-4=9.a=-2 xt: b-2a=1=9b=-3 x": -2b=6 is consistent with the value found above. COMMENT The final coefficient comparison is useful for checking the validity of the solution. Always be thorough and compare all coefficients, even if you do not write down the check as part of your onswer. oof (x)=(x-2)(x?-2x-3) =(x-2)(x-3)(x+1) (©) Equal roots when A=0: b—4ac=0 (k+1))-4x2kx1=0 kK -6k+1=0 n k S20 sani Positive cubic shape. BD) No real roots when A0 for all values of x, ‘The numerator of the rational function is never equal to zero, hence y #0. p93 Poni 28 Vertical asymptote where denominator @ f(x)=x'-+x° +x? +41 is zero: x=—-2 a f(I=141414141=5 b (e-i)s(x)=(x-1)(x! +x° +x? Fx41) (x+2)y=x? 44x45 xt +(4-y)x+(5-2y)=0 (ey¥=a@-2) axttxteextex —4)* \(4—y) -4(5-2 (v-4)# y(4-y) -46-29) (x2 tax) -1 y-intercept at -1. ‘Quintic shape: graph of y= x° For real solutions of x, require translated down one unit. yr-420 y ie y24 yS-2 or y22 Vertical asymptote at Range excludes the interval }-2, 2[ : (e421 aay4t x+2 x+2 As x9 00, y > x+2, so the line y=x+2 isan oblique asymptote. Figure 3ML.4 Graph of y = x°-1 4 From Figure 3ML.4, y= x°—1 has a single real root at x =1, and so (x-1) f(x) has a single factor (x~1). ‘Therefore f (x) has no linear factors and hence, by the factor theorem, Jf (x)#0 for any value x. Figure 3ML.3 Graph of y= ~ "> COMMENT Finding oblique asymplotes is not fexpecied within the syllabus; plottin, rational functions is covered in depth in Section SF. Exercise o- =-i46 (3x-1)""=1 64+ =0 3x-1=1 a 2 (a*)'-6a" +5=0 or 3x— 1 and x'—4iseven or x*-4=0 and 3x-140 ean? or x=0 or x=42 8 x([x|-4)=0 =x=0 or |x|=4 0 or x=44 Exercise GED © 9(1+9**)=10x3* 9 49-10x3"=0 (3° 103") +9=0 Let u=3*: (log, x)’ log, x+5=0 Let u=log, x: w—6u+5=0 (u-1)(u-5)=0 —10u+9=0 u=1 or u=5 (u-1)(u-9)=0 hel or woo cvlog,x=1 or log,x=5 wx=2'=2 or x=2'=32 or 3* COMMENT Instead of changing log, into log, the ‘opposite could have been done, o both log, and log, could have been changed to log {base 10). 32 Inx ceases to be defined at x= 0; as x0, xln x0, so there is no vertical asymptote, but there is an empty circle at the origin and no graph for x <0. ‘As x gets large, both x and Inx continue to increase, so there is no horizontal asymptote. xInx=0when x=0 or In x=0. The first root has already been eliminated, so the only root is x ¥ x (0.368, -0.368) Figure 4C.2 ine Inx=0 at x= 1, s0 there isa vertical asymptote at x= 1. Inx ceases to be defined at x= 0, and as x90 from above, Inx > ~=», so the graph terminates with an empty circle at the origin. For large x, e* increases more rapidly than In x, so their ratio increases and there is no horizontal asymptote. y= 0when e* =0, which has no solutions, so there are no roots. “The exact value of the minimum is best found using a GDG, but from the above we can be confident that there must be a single minimum at some x > 1. Topic 4G Features of graphs e e €* #0, so there is no vertical asymptote, For large positive x, e* increases more rapidly than any polynomial in x; so as xe, y90, For large negative x, e* > 0 and the numerator is positive, so ys, =O when x=0 (double root) or x=33. Exact values for local minima and maxima are best found using a GDC, but from the above we can be confident that there must be a minimum in ]-3, O[ and one in ]0, 3[, and maxima at the origin and in }3, ef. y (6.07, 2.70) (1.42, -3.40) (not to scale) (250,209) Figure 4¢.4 Exercise Gi xlnx=3-37 ‘The graphs of y=xlnx and y=3-x? intersect at one point. From GDC: x= 1.53 (3SP) For Inx =ke to have exactly one solution, the graph of y= ke must be tangent to the graph of y=In x. y Figure 4D.3 Graphs of y= kxand y=Inx Ing? =2Insx, 50 Inx® =o is equivalent to Inx= 4s, which will have two solutions since the line y= fx hasa smaller gradient than y= kx. u(2)-m Inx, so x (2) = ke is equivalent to Inx =—kx, x which will have one solution, since there will be an intersection in the lower right quadrant. Inge=Ina! =4Inx, so ing =x is equivalent to Inx = 2kx, which will have no solutions since the line y=2kx has a greater gradient than y= Exercise © Substitute y=8—x into the circle equation to find the intersections: x? -6x+(8—x)—2(8-x)+2=0 2x*-20x+50=0 x? -10x+25=0 (x-5)' <0 ‘There is a single solution (5, 3), so the line is a tangent to the circle. comm Instead of solving the equation, it would be reasonable to show that the discriminant is zero and hence conclude that there is only one solution. In this case, itis easy enough to show that the quadratic is a perfect square, so solving the equation is just as efficient. (©) Substitute the equation of the line into the quadratic equation to find the equation governing intersections: 3xt—x+5=mx+3 = 3x?-(m+1)x+2=0 For the line to be tangent to the curve, this equation must have a single solution, so discriminant A=0: (m+1)-24=0 m+1=22V6 =-142V6 From the equation of the line, By=k-2x, 0 9y* =(k—2x)? ‘Substitute this into the ellipse equation to find the equation governing intersections: 4x? +(k-2x)' =36 8x? —4kx +k’ -36=0 4 Algebroie structures 33 p= For the line to be tangent to the ellipse, this equation must have a single solution, so discriminant A=0: (4k)? -32(k?-36)=0 16k" +32x36=0 Ke =2x36 k=t6V2 COMMENT When calculating volves parhvay through solving an equation, it is offen needless work to multiply out products if you are subsequently going to divide through by ‘2 common factor. In his case, rather than evaluating 32 x 36 itis more convenient to leave it in product form, ready to divide through by 16. © intersections when ket5= 2742 x? —kx-3=0 Discriminant A = k? + 12 > 0 for all values of k, so the quadratic has two roots, ice. there are always two intersection points, for any value of k. Substituting y= 4—. from the second equation into the first equation: 2° +2!* =10 2-10 x2 +24 =0 ‘This isa hidden quadratic. Let u=2%; then w—10u+16=0 (u-2)(u-8)=0 u=2 or u=8 ie 2*=2 or 2=8 >x=1 or 3 (x, y)=(3) or (3,1) 34 Topic 4F Systems of linear equations © Substitute y=.x° into log, x+log, y=3: log, x-+log, (x*)=3 x-2y- z=2. Bx+4y-3z=0 |. Eliminate y from (2) and (3): @ 2xty-22=0 (1) (2)#2xQ) Sx 5252 ...(2) (3)-4x(1) 5x +5z=0 «..(3) For (4) and (5) to be consistent, require c=-2, ©) x-2yz22=0 0) 2xtky — we (2) xm yt3e= 3) Eliminate x from (2) and (3): () x =2y422=0 (2)-2x(1) (k+4)y—5z=3 (@)-@ Eliminate z from (4): yte (i) x -2yt22= ol) (4)+5x(5) (k+9)y 22 ...(6) 6) yt 22-5 ...(5) From (6), y= so there is no valid unique solution when k=~9. Gxt y- 2-2.) x-2yt2z=1 ant y-4zea ...(3) Eliminate y from (2) and (3): @) 2xty- 222 (2)#2x(1) 5x =5 (3)-@) ~3e=0-2 The solution is Eliminate x from (2) and (3): a) x-2y+ z= (2)-ax() 5y-3e=a-2 (3)-4x(1) 15y-92=0°—4 Eliminate y from (5): (Q) 0 x-2y4 251 () Sy-3e=a-2 (A) (5)-3x(4) 0=a°-4-3(a-2) ...(6) For a consistent solution, require @-3a+2=0 (a-1)(a-2)=0 a=1 or 2 > With a xo dy 5y-32=0 Let 2=5t; then yo3t and x=1+2y-z=1+t general solution is (x, ys2)=(I+ 1,34, 5¢) COMMENT When parameterising, it can be convenient fo use judgement to avoid fractions in the end solution. IF we parometerise as z= then the solution would be (Eb) which is equally valid but less tidy. ha x-2y+ 2=7 Qxt y-32=b ...(2) xt ytke=4 ...3) Eliminate x from (2) and (3): (Qo x-2yt 0 ze7 (2)-2x(1) Sy- Se=b-14 (3)-() By #(k-1)e=-3 Eliminate y from (5): (a) xn 2yt z=7 (4) Sy- Sz=b-14 5x(5)—3x(4) (Sk+10)z=27-3b ... 3(9-b) 5(k+2) there is no unique solution, From (6), z= +80 for k=-2 b For k=~2 the system will be consistent if 27-3b=0, ie. © With k=-2 and b=9: x-2yt 2=7 Sy-5e=-5 Let z=#; then (x,y z)=(t+5,t-L4) 4 Algebraic structures Figure 46.2.1 Graphs of y= x and y= 5Inx From the graph on GDC, x>5Inx for 0127 Figure 4G.2.2 Graphs of y= 5 ond ad From the graph on GDC, ->5 for Inx 112.7 The answers are different because for x €]0,1[, Inx<0, s0 for x<1 the conditions (a) and (b) are opposite, whereas for x>1 the conditions are equivalent. “ 36 Mixed examination practice 4 Exercise B+ prt g=(x-a)'(x-b) =(x?=2ax-+0°)(x-b) =x°+(-2a-b)x? +(2ab+a") xa" Comparing coefficients: wel : 0=-2a—b => b=-2a xh p=2abta?=—3a° ie. 4p’+27q" =0 Mixed examination practice 4 Short questions © 2 y=2*: axis intercept at (0, 1), ‘exponential shape, y=1-x*: axis intercepts at (0, 1) and (41, 0), negative quadratic. y= Figure 4MS.1 b Two intersection points => two solutions of 2* =1—x?, ©) sketch the graphs y, =x?—4x and y, =e” {Sketching the graph on GDC: y (2, 0.541) Figure 4MS.2 Maximum value of y is 0.541 (3SF) on GDC: y yizat de Figure 4MS.3 ‘The intersections are at x=-112, -0379, 2.02 (3SF) N1>Y, for x€]=1.12,-0.379[U]2.02, [ 3x- y+22=2 ...() x-2y+ 253...) x y43z=-5 ...(3) i Eliminate x from (1) and ( (i)-3x(2) 5y- z QQ) x-2yt z (3)-(2) yt2z=-8 Eliminate y from (4): (4)-5x (5) -11z=33 (2) x 2yt . (3) yt2e=-8 So the solution is 3 ya-8-22=-2 x=3+2y-2=2 ie, (x,9,2)=(2,-2,-3) ef Inx = 3e* e(Inx=3)=0 e*=0 (no solutions) or Inx=3 = Bo x1436=13%" (2? -13x7 +36=0 (u-9)(u-4)=0 u=9 or u=4 wx=4 or 9 x=42, £3 y=x'—13x" +36 is a positive quartic with four roots, so y <0 between the first and second roots and between the third and fourth roots. ‘That is, x! + 36 < 13x? for xe[-3, -2]U[2, 3] Algebraic structures . @ © Vertical asymptote where Substitute y=2x—k into the circle : ef =2=9x=In2 equation to find intersections: ‘As xe», denominator gets very large > > and positive, so y+0 24 (Qx-b) #5 As x —e°, denominator tends to -2 Sx? —4kx +k? =! so yo} Ifthe line is tangent to the circle, then Numeratoris never zero, so 40 there isa single solution to this quadratic Wa 4 equation, so the discriminant A= 0: (-4k)'-20(k?-5)=0 -4k? +100=0 R=25 k=s5 (© On GDC, sketch graphs y, “5 )z=In2 4 Figure 4MS.7 x=In2 Qxt2y- z=1 (I) xt yo met 4x+4y—32=p (3) Eliminate x from (1) and (3): ()-2x(2) (2) +2) Figure 4MS.10 (3)-4x(2) 2=pt6 ...(5) ‘The intersections are at x=1.43, 8.61 For this to be a consistent system, need ‘The graph of y, * has a vertical inx asymptote at x. VSI for x€]0,1[ 1.43, 8.61[ Let y=#; then. elas yrz)=(5—t, 69) D Mixed exasination procice A , ©) xinx+4inx=0 (x+4)inx=0 x=-4 or Inx=0 “eal (as Inx has no real value for x=~4) COMMENT ‘Always check the validity of algebraic solutions, especially in functions with restricted domains, such as rational functions and those containing logarithms. Figure 4ML.1.1 ii, Vertical asymptote x =4, horizontal Long questions asymptote y=0. © «i Expanding: Y y=(e-3tne (x-a)’-b=x° -3ax?+3a?x—-a°—b ‘Comparing coefficients: (1.44, -1.38) ‘i With k=27, the equation Figure 4ML.1.2 Gueee. 28=0 is From GDC graph, the minimum int is (x, y)=(1.44, -1.38) (e-3)'<1 point is(x, y)=( ). Let u= x’; then equation becomes sxa4 w+u-6=0 Vertical asymptote at x: (u-2)(u+3)=0 For values close to x=3, yis very u=2 or u=-3 Leeda? sx?=2 (rejectx?=-3) As x +05 denominator gets large and positive so y—+0 from above. 80 x= 42 ‘As x 909 denominator gets large and negative so y 0 from below. axt yt zs3 xtky+2z=4 ...(2) xk) +(x+ky 6 x- y+32=b ...() ax'+dke 46x 44k x tk Eliminate x from (2) and (3): 4x7 42k +h —6 Q) xt yt 253 x! take! +(6k? +1)x" @)-Q) (ky =1 + (4k +2k)x+(k! +k -6) ()-() -2y+22=b-3 Eliminate z from (5): COMMENT To expand (x+k}* quickly, use the (QQ) xt yt 253 binomial theorem; see Chapter 8. (ey =1 (5)-2x(4) -2ky =b-5 From (6), if k=0 then there is no unique solution. ‘Comparing coefficients: x: 7 = 6K? +1 is consistent with If} =5 then equation (6) is valid as = 4k? +2k is consistent with 0=0 and the system is consistent. =k +R —6 is consistent With k= 0 and b=5, the system reduces to So for k=—1 the two equations are xtytza3 equivalent. -ytze1 Let ‘Then y=-1x=3-y-z=4-t S0(x, y.2)=(4—tt-1,t) ii From (b)(i), x4+7x? =4x° +644 is equivalent to f(x-1)=0. From (a), f(x1)=0=3x-1=4V2 sxsltv2 c x'+7x? > 4x? +6x+4 is equivalent to f(x-1)>0. y= (2-1) isa positive quartic with two roots at x=1+ V2; it is positive on either side of those two roots, ie. for x1+V2. The theory of functions Exercise F(x)=¥ln(x—4) ‘Square root can have only non-negative values in its domain, so require In(x—4)20: x42" x25 Domain of f(x) is x25 Fu +2 (x-3)(x-2) 47 41 OB fw- Cannot have division by zero, so x# 2, 2,3 Square root can only have non-negative values in its domain, so require x~ 120, iex21 Domain of f(x) isx21x#2.x#3 COMMENT Note that the restriction x #-2 is not needed in the final answer as itis already covered by the restriction x2 1. Require that the boundary at x=2 be consistent in the two parts of the function: 3x2? -1=a-2? ll=a-4 .a=15 © ox)=in(x? 43x42) Inx can have only positive values in its domain, so require x’ +3x+2>0: x 43x+2>0 (x+(x42)>0 x<-2 or x>-1 Domain of g(x)isx<~2orx>-1 COMMENT In may be helpful o draw o graph of y =x! +3x42 to solve the quadratic inequality x?+3x+2>0, Bx —4 O sw- (E34 Cannot have division by zero =9 x #12 Square root can have only non-negative values in its domain, so require either 8x-420 and x-12>0 or 8x—4<0 and x-12<0. 8x—420 and x-12>0 ax2y and x>12 sexo 8x-450 and x-12<0 axst and x<12 sxst So domain of f(x) iss or x>12 £5 The hoor of inci (f= Vx-a+in(b-x) 1 Square root can have only non-negative values in its domain, so require x>a Inx can have only positive values in its domain, so require xb= function has empty domain Ja=a+In(b-a) ifa 0, so the domain of frisx>0. Balx)=2{x?)43 yar $3, RYO=4 pS © @ To find the inverse function of fla=e": yoe™ 2Qx=Iny sxetin iy n F'G@)=Zinx=Inde ‘To find the inverse function of g(x)=xth: yextl spxsy-l ag t(x)=x-1 So £°(3)xg"(3)=(InV3)x@-1) =2Iny3 =In3 > (fe) @)= 7S 7(@)= Inve 2. (gy'@)=Iny3-1 DO P=" (20 > 4° =0.25=x=-1 Oo axin4 reer) (x7 +9) y=x2-4 x yt9yax*—4 xy-x? =-4-9y 2x?(y-1)=-4-9y 449y (as domain of fis x <0) Hence f"'(x)= “The graph of fhas a horizontal asymptote aty = 1 (as x->—e) and is decreasing for all x $0, so the range of fis ye I. 2 SE.11 Graph of f{x)=~5— Figure roph of Fx) = "== real x Hence the domain of fis xe [- a f(x)ax7,xSk ‘Taking k=0, y=x? =px=—y/y (choose the negative root since the domain of fis x <0) of" f(a)=(x+1) +2, x2k Taking k=-1, y=(x41l +2 (x41 =y-2 x+1= fy=2 (take positive root since domain of f isx2-1) sexs y-2-1 2 fl(x)=Vx-2-1 S(x)=|a]. xSk Taking k=0, x (since x $0) ‘The range of fis R, so the domain of FtisalsoR. b f(x)=In(x—1)+In(3) =In[(x-1)) =In(3x-3) sgf(x)aee =3x-3 _| 24-1", x2 Finding the inverse of f(x)= 1 =x=— x 1 1 ef aet x So f'(x)=—= f(x) self-inverse, tndis 7 3x-5, Finding the inverse of g(x)=""—?: xtk ylatk)=3x-5 wy thy=3x-5 3x-ay=S-+hy x(3-y)=5+hy Require that g(x) ae Box rkx-5 x3 ‘Comparing these, it is evident that k= IF itis difficult to see that multiplying the numerator and denominator os above enables straightforward comparison to dotermine k, then the following (more lengthy!) process can be undertaken instead: a St he xtk (3x—3)(3-x)=(5+kx)(x+k) 9x-3x? -15 + 5x= Sx + Sk + bx? +x 3x2 41-15 = be? + (12 45) x4 5k Comparing coefficients of the two sides: x7: Bak x 14245 2°: -15=5k ‘These three equations consistently give the unique solution k=~3. Exercise (ij _3x=1 DY 45x Vertical asymptote where denominator equals zero: x= $ Horizontal asymptote as x» tee: L By Gfe=35 a Cannot have division by zero, so domain is x#~-3 Figure 5.6 Range is y#0 acta by a8 y(x43)=1 Vertical asymptote where denominator equals zero: x =5 Horizontal asymptote as x gets large: y= 3 Axis intercepts: (a 2) and G 9) ¥ P B se) 28 ee Be a Horizontal asymptote as x gets large: a Figure 5F.8 Groph of (x)= positive a Range of fis yeR, yey —oxt3 y 2x-8 (The domain of fis the range of f) For fto be self-inverse, require that S(x)= F(x) for all x. ‘The vertical asymptote of fis x= 45 this must be the same as the vertical asymptote of f~!, which is x =4=a=8 5 the theory of functions aes (Ay ® Mixed examination practice 5 bp ya teh : 5x Short questions (6-x)y=4x-3 a y=log,(x+3) Sy-ay=4x-3 Vaxt3 AxtxyaSy+3 x=37-3 af (x)=3-3 (Range of fis R, so domain of fis x(4+y)=Sy+3 appeerts, yt so R..) 1 5x43 ol 1 FG) EB b ya3e"4 * Jag Oe f@)=x7-6x+10 a =3) 9410 n(2)=-1 =3) 41 a-(is(2)} » raat x-3=Jy-1 4 x) (the positive square root is needed as x 23) setae teu(®) (Range of 7 ‘domain of SSH} inge of gis y > 0, so domain of gis i" x>0) . a fi(x)=3+VeaT © The minimum point of fis (3, 1), so the range of fis y2 1 and hence the domain of f-is x21. © @ Reflection of f(x) in the line y= x gives the graph of f'(x), so Cis y=log, x. b Cuts the x-axis where y = 0: cats the x-axis where y 2 h(x)=x*—6x42 Tegse=o =(x-3)'-942 > =(x-3)'-7 i.e. intersection at (1, 0). a Vertical asymptote where denominator equals zero: x=5 Horizontal asymptote for large x: yoto ‘The domain of h is x > 3, so the range ofhis y>—7 h(x)=(x-3)'-7, x>k For the function to be one-to-one, take k=3 ya(x-3P “7 (x-3) = y +7 x-3= yr (choose the positive root since the domain is x >3) sxo3+\y+7 oW(x)=34vx+7 Horizontal asymptote is y = -1, so the range is y#—1. Vertical asymptote: x = Axis intercepts: (3, 0) and (0, 3) o | c yedk Yer (x+1)y=3-x =x43 x+1 fw) Domain and range of fare x#-Landy#-1, so domain and range of fare x#-Land y#-1, Sox, x<0 pet, x20 a i Withp=3: lower part has range ]5,°[ upper part has range J0,3] ws f (x) has range ]0,3] U ]5, 09[ ii For x<0: yaS-x=x=5-y of (x)=5-« For x20: ya3e* 3 7 y =x=h(2) y o Ftayein(2)w 3 Inj)—], O3) of a)=Ve=1 ii Domain of fis x >3, so range of f" is y>3. ili’ Range of fis y>10, so domain of f! isx>10. F(x)=8(3x) x e1=5-3x x? +3x-4=0 (x+4)(x-1)=0 x=-4 or x=1 However, the domain of fis x>3 50 there are no solutions to this equation. i f(7)=2x7+1215 ii Range of fis R i = (3 x-1 iv ff(x)=2(2x+1)41 = 4x43 b The value f (0)=1 isin the range of f but not in the domain of g, so gf(0) is not defined. ci yt (x-1y=x43 i g(x) is self-inverse, so the domain and range of g(x) and the domain and range of g-'(x) must all be the same. domain of g(x) is x# 1. ‘ill Range of g(x) is y #1. a f(x)ax?+4x49 = (x42) 449 =(x+2) +5 © Symmetry line at x=-2, vertex at (-2, 5). Positive quadratic shape; y-intercept at (0, 9). (2, 5) Figure 5ML3 Range of f(x) is 15,01 Range of g(x)=e* is ]0, -[ A(x)= feg() Range of h(x) is the range of f(x) with restricted domain 10, 2 range of h(x) is ]9,20[ (2x+3)(4-y)=12 8x+12-y(2x+3)=12 y(2x+3)=8x 8x meee ras) Vertical asymptote where denominator 3 Is zero: x=—— equals zero: x=—> Horizontal asymptote: y Single axis intercept at (0, 0) 8x Figure 5ML4 Graph of y= 5 Thetheory of functions g8(x)=8(a(x)) = 824k) =<") 22x+k)+3 - _ 16x 48k =(x') x+2k+3 _ ii Vertical asymptote where b sesyeas(2}=ace (0) denominator equals zero: J" ek Replacing x with —: ~ x A }e2p=21 (2) © (1)-2x(2): ~3f(x)=2x- 1 4aas = fle)=—P _Atx-2x? Finding the inverse: oR _ 16x76 7 gen 16 (4x-16)y=16x-76 Axy-16y=16x-76 Axy-16x=16)-76 (4y-16)x=16y-76 ie. hg(x) is self-inverse. 52 Mixed examinton practice 5 Transformations of graphs Exercise Go y Figure 6D.6 Graphs of y =|3x-+1| and y=2x Brom the graph, y, > y, forall x, so the solution isx €R. 0472 8.47 Figure 6D.7 Graphs of y =|x”~ 4land y=8x Intersection of y= 4—x* and y=8x in the interval (0, 2]: 4x? =8x 2 +8x-4=0 84 V64416 2 8445 “2 =2V5~—4 (discard negative root) Intersection of y= x?—4 and y= 8x in the interval [2, x 4=8x x? =8x-4=0 8 J64+16 2 _8t4y5 2 =2V5~4 (discard negative root) s. the solutions are x=2V5 4= 0.472 and 847 |x? -7x+10|=-(x?-7x+10) 9x°-7x41050 =>(x-5)(x-2)<0 A positive quadratic is negative between its two roots Puy PR Pe WD le+al-e-20 COMMENT This question can be solved either using «a graph and algebra by intervals or by direct algebraic calculation. Both methods are given here, and either would be acceptable in an examination. Graphically: napa Y Figure 6D.10 Graphs ofy =|x+2"| ‘and y=|x-244 Intersection of y= x+-q’ and y=2q" xin the interval -q’, 24° xtq=2q'—x 2 4 axet ry Algebraically: (x+q?) =(x-29°)' x? +2xq? +g! =x? —4xq" +4q* 6xq? = 34" 2 4 sxek a & 54 _ Topic 6 Consecutive transformations y= fo)+| FO) y= 0 wherever f(x) 3 Sulx)=a(-3x+1)-b-2 =-Bax+a-b-2 s.g(x)=4-15x=a-b-2-3ax ‘Comparing coefficients of x': 3a=-15 = a=5 ‘Comparing coefficients of x°: a-b-2=4 => b=-1 (x)= ax? thx Reflection in x= 0: replace x with «=> falx)=ax? —bx-+e pn. 1 ‘Translation »( 3 } replace ewith (x1), add 3 => fx) =a(x41)'—b(x41) +043 Horizontal stretch with scale factor 2: ahe replace x with = => 2 Es) -o(Zsr}sers 2 2 a. b sox" +axta——x—b+c+3 4 2 =2ta(aPebanbecr3 4 2 oig(x)=4x" +ax-6 afaa(a-2)etanbeers 4 2 ‘Comparing coefficients of x*: #245 a=16 4 ‘Comparing coefficients of x': 6 a-2=a= b=0 2 Comparing coefficients of x* a—b+c+3=-6 = c=-25 OD sl)=2+x Vertical stretch with scale factor 8: ‘multiply through by 8 => fi(x)=8(2*+x) 1 ‘Translation by (;}: replace x with (x=1), add 4 = f,(x)=8(2" +x-1)+4 Horizontal stretch with scale factor +: replace x with 2x => 2 file)=8(2" 424-1) +4 x21 4162-4 = 416x—4 =(2)"" +164 =a 416x—4 Soh(x)=4*!+16x-4 a Graph of y=Inx: vertical asymptote y intercept (1, 0) Figure 6E.12.1 b Graph of y=3 In(x+2)is obtained from the graph of y=Inx 2 translation ( ° ) and vertical stretch with scale factor 3 = vertical asymptote x =—2; intercept (-1, 0) ¥y 25 ne+2) Figure 6E.12.2 ) 4.Transfermations of grophs 55 i © Graph of y=In(2x—1)is obtained from the graph of y =Inx by: translation and horizontal stretch with scale factor 2 = vertical asymptote x= } intercept (1, 0). Figure 6£.12.3 Exercise ; © 2 F(x) to f(x-3)is a translation »()] 2 p=3,q=0 b i. Thisis the graph of y= x’ translated 3 () x-intercept (and vertex) at (3, 0); yr-intercept at (0, 9) y Figure 6F.3.1 ic 6F Reciprocal ransfrmations, ‘his is the reciprocal transformation of (i): + vertical asymptote atx =3 © y>Oasx—teo + pinta) y 1 F Y= Fy mila: + vertical asymptotes where f(x)=0: x=-2andx=1 ‘+ amaximum at the minimum of f(x): approximately at (8 +) + aminimum at the maximum of f(x), (1.8, -}) {© Reciprocal graph followed by 0 translation _ } + no vertical asymptotes as g(x) #0 + maximum at (1, -1) + roots of 1 at asymptotes of g(x) g(x), Le. at (0, 0) and (2, 0), so new curve passes through (0, -2) and (2,-2) (B) Vertical asymptotes where denominator equals zero: we" 4x? =0 <(e"-4)=0 x=0 ore" =4 s.x=0 or x=tvind Roots of L0) are at roots of f(x): (1.5, 0) g(x) Asymptotes of £ are at roots of g(x): glx) x=Oandx=5 As x00, f(x) is negative and g(x) gets large and negative, so) from above. g(x) Figure 62.7 Exercise (© a The graph of an odd function has two- fold rotational symmetry about the origin. b i Even function = reflective symmetry about x=0 y Figure 66.2.1 ii, Odd function = two-fold rotational symmetry about the origin y Figure 66.2.2 ) 4 Transformations of grophs 57 yk a BD Fane (1+(-x)') =e" (144) =s(z) So by the definition of an even function, f(x) is even. ©) Foran even function, f(-x)= f(x), so it cannot be one-to-one as each value in the range has at least two corresponding values in the domain. Suppose that f (2x) and g(x) are both even functions, and leth(x)= f(x)+ g(x) for x€Dom, =Dom,™Dom,, Then h{-x)= f(x) +g(-x) =f(x)+g(x) because f and g are even =h(s) (x) =h(x) for all xin its domain, so by definition h( © suppose that f(x)and g(x) are both odd functions, and let h(x) = f(x)x g(x) for x€Dom, =Dom,Dom,. Then h(-x)= f (-x)xa(-x) =(-F(x))x(-g(x)) because f and g are odd =f (x)xa(x) h(x) ¢- h(-x) =x) for all xin its domain, so by definition h(x) is even. (For f(x) to be odd, require that f (—x)=—f(x) for allx€Dom,. 2 If f(x)=ax" is odd, then a(x)" =-ax" ‘One solution is a=0, but this gives a trivial function f(x)=0, which is not of interest. wen) =x" For real-valued f, it must be the case that m is an integer. ‘Then the above condition becomes (Ay'x"=-2" forall xe Dom, (ayt=4 which is true when 1 is an odd integer and false when m is an even integer. emis odd. b 58 Topic 6G Symmetries of graphs and fun G Suppose that f(x) is an odd function and g(x) is an even function, and let (x)= sf (2). ‘Then for all x€ Dom,, n(-x)=a(f(-*)) =g(-f(x)) sincef is odd =(f(x)) sinceg iseven =h(x) e.h(-x) = h(x) for all x in its domain, so by definition h(x)is even, O: £ 46x47 =(x+3)'-2 has line of symmetry x=~3 b> f(%=a)is the function f(x) after a translation by| (*) Require a symmetry line atx =0 for f(x=a) tobe even. 1a=3 ‘The graph has line of symmetry x =5 since f(5+a)= f(10-(5+a))= (5a) D ta function J (x)issymmetrical in y=x, then f (x)= f-'(x) since the graph of f(x) is the graph of f(x) reflected through y=zx. © F(x) = for all x and hence ff(4)=4 © Let g(x)=$(f(4)- f(a). Then at-x)=HF-8)~Fla)) =-8(*) +:8(-x)=—g (x) for all x in its domain, 50 by definition g(x) is odd. P b By similar reasoning, W(x) =(f(4)+ f(x) isan even function: Wax) = (F(x) +8) AC) For any function f, F)=F(/)-se-9)) +49) = g(x)+h(x) ive. f(x) can be written as the sum of an odd function g(x) and an even function h(x). i Lie_ex di s)=5(e -e*) Axis intercept at (0, 0) only. y Figure 6.12.1 Verges ii a(x)=(e +e*) Axis intercept at (0, 1) only. y 64 Transformations of graphs COMMENT These are the hyperbolic sine and cosine functions, g(x) = sithx and h(x which you can find on your calculator. If fis an odd function, then f(-x)==f (x) for all xin its domain, Iffis also a polynomial, then since any polynomial is defined at all real values of x; it must be defined at x=0. f(-0)=-F(0) because fis odd, but also f(-0)= f(0). So f(0)=-F(0) and hence f(0)=0, which means that the graph of f(x) passes through the origin. ‘This must always be the case when 0 is within the domain of f(x). ‘An example of an odd function not defined at 0 would be f(x)=x", or indeed f (x)= x" for any positive odd integer n. ‘f(x)=cotx and f(x)=cscx are other ‘examples encountered in this course. Mixed examination practice 6 Short questions ©) a Graph of y=3 f(x-2) is obtained from the graph of y= f(x) by: 2 translation| |and vertical stretch with scale factor 3 => asymptote becomes x= 0; x-intercepts become (2, 0) and (6, 0) ‘Mixed examination practice 6 Figure 6MS.1.1 b Asymptotes 75 at roots of f(x): x=Oandx=4, y0 from below asx—>—soand 0 from above as x —> <2, ‘Maximum value where f (x) has minimum. y x (1. Figure 6MS.1.2 f(x)=x-1 ‘Translation by (°}: replace x with (x-2)= f,(x)=(x-2)-1 Vertical stretch with scale factor 2: multiply by 2 f,(x)=2[(x-2)-1] So new graph is y=2(x-2)'-1] = 2[x?-6x? +12x-8-1] = 2x°-12x7+24x-18 Figure 6MS.3. Graphs ofy =|2x-1| and yex Intersection of y=|2x~1| and y=x in 1 interval <>: 1-2x=. axst 3 Intersection of y=|2x—1] and y=. in interval x> 3 2x-l=x sltx=a f= > _3(x=2)+1 4-2 _3(x=2) x2 =o x2 .p=3,q=1 1 g(x) =4, then f(x)=3+ (#-2) ‘Transformation from g to fisa translation »(3) 6 Transformations of graphs Range of f(x)is y #3, so domain of f-'(x)is x43. © The graph of y= f(x)is obtained from the graph of y= f(x) by reflecting in the line y=. a Translation »(3) b i y=In(x+2)is the graph of y=Inx after translation by (0) 1 y= T asa) of y=In(x-+2) vertical asymptote where In(x+2): = asx—>-2, y->0 from below as x02, 90 from above is the reciprocal graph yeintercept at 1, no x-intercept inz yan +2) =(n@ +2) Figure 6ML.3.1 Graphs of 1 y= W(x+2)andy = oy {The turning point on the right has shifted from (1, 4) to (3,~4). Reflection in the x-axis has switched the sign of the y-coordinate, so the 2 translation must be| ( } ified ination erties a si g(x)has been translated »(3) and reflected in the x-axis, so H(x)=—g(x-2). H(x)=-g(x-2) =-((x-2)'-2(x-2)+5) =-(x° 6x? 412x-8-2x+445) =x? 46x" ~10x=1 1, b=6, c=-10, d: i Roots of y=(k(x))’ are at the same values as shown for y= k(x). K(x) appears linear close to the roots, so the square of the curve should look quadratic close to the roots. y Figure 6ML.3.2 COMMENT IF you recognise the given curve as k(x) = 2sin| "you will have a shoricut to working out the shape of the result: (kin)? = si (TE) =2-cosne, This material is covered in Chapter 1 cahough recognising the curve i it is not necessary. Pray Re o i f(x)=x? -7x+10=(x-2)(x-5) Positive quadratic with roots at 2 and 5; yrintercept (0, 10), Figure 6ML4.1 2 b f(x{)=(x/)'-7xl+10 —7|x|+10 = a(x) The modulus transformation replaces xby|x|, so the graph for negative x is just the mirror image (reflection in the y-axis) of the graph for positive x. y y=at— Ta] +10 Figure 6ML.4.2 ad x?=7|x|+10=27 Nid. e x*-7|x|+10=-2 x°-7|x|+12=0 (/-3)(1-4)=0 |x|=3 or 4 x=d3,4 Line of symmetry atx=3=> f(x)=a(x-3) +e Expanding: ax? —6ax+9a+c= 3x? +bx+10 Comparing coefficients: ~6a=b=9b=-18 x°: 9atc=10=>¢=-17 1b=-18 f(x)is symmetrical about x=3, so fore £3-k) Replacing k with x—3 gives the other form of the symmetry condition: f(3+(x-3))= f(3-@-3)) F(x)=f(6-x) 6 g(x)= f(x+p)+qis.a quadratic; itis even and goes through the origin, so its vertex is at the origin. ‘g(x)has symmetry line atx=Oand ‘p-value of vertex raised from c=—17 to 0, so it is obtained from f(x)by a 3 translation| 7 = g(x)= f(x+3)417 .p=3,q=I7 Since g(x) is an even function, by definition g(x)= g(|x|) for all xeR. (4s. 4Transformotions of graphs ii, Horizontal modulus transformation: 0 the graph for negative x is the mirror graph of y=e* translated »(3) image (reflection in the y-axis) of the graph in (a) for positive x. Horizontal asymptote at y=—2 for y large negative x; intercepts at (0, ~1) and(In2, 0). ‘The graph of y=e* ~2is the y Figure 6ML.6.3 Require those values of x for which the Figure 6ML.6.1 graphs in (b)(i) and (b)(ii) coincide: The graphs are the same for x > In 2 ‘They intersect for the negative value of x where e*-2-e* e*—4e*+1=0 ea24 V3 in (223) Since this should be a negative value of x, the intersection is x=In (2-3) bi. Vertical modulus transformation: ‘graph in (a) has its negative-y part reflected in the x-axis. SS aa iicel exsision omc 1) 3 4 er Exercise Bo 4 =3(2)-20)=4 u, =3(4)-2(2)=8 4, =3(8)-2(4)=16 b i Itappears that , =2" ii Mu, =2" then u,_,=2"* and =2"(3-1) =2"x2 bat "satisfies the equation comm This is an example of o method called proof by induction, which in this case can be used fo establish that the result v, = 2° is tue for ollne z. This method of proof is covered in Chapter 25. 7 seauences and series Exercise @ a a=5,a,213 b a, <400 8n-3<400 nS = 50375 So the first 50 terms are less than 400. © 4=61 sou +(10-1)d=61 =u,+9d=61 (1) y= 79 su +(13-1)d=79 =u, +12d=79 ...(2) (2)—(Ds 3d=18=>d=6 yy = Myp + 10d =61+60=121 COMMENT Here 10d hos been added to the tenth term to find the twentieth term, but this could also be colculated by firs! finding 1 from equation (1) or (2) and then using the usval formula for v,. ou, +(8-1)d=74 =u47d=74 —.l) s2td=a-b 4, =137 uy =2a+b+7 st, #(1S“1)d=137 e242d=2a+b+7 =>u,+14d=137 ...(2) (2)-(): 7d=63 => d=9 Substituting in (1): ,+7X9=74 => u,=11 Let u, =2275 then Substituting d=—b in (1): 2-b=a-b d=a-3b-(2a+b+7) 2.-b=-a-4b-7 (© Third rung is 70cm above ground: ~a-7 4, =70 =b= wu, +(3-1)d=70 =u,42d=70 ...(1) ‘Tenth rung is 210 cm above ground: oe Hq = 210 digits for a total of 9 digits. my +(10-1)d=210 10th and 11th pages are each =m +9d=210 ...(2) numbered with two digits, @-a): ‘Total number of digits for the first 7d=140 = d=20 11 pages is 94+22=13. Substituting in (1): . a 4.2%20=70 => 1 =30 First 9 pages: 9 pages at 1 digit per ‘Top rung is 350cm above ground, age: total 9 a ios i oo Pages 10-99: 90 pages at 2 digits per 30+(n—1)20=350 page: total 180 50n-410-=350 So the total number of digits on the 300 first 99 pages is 189. aan! ‘Then, since pages 100-999 have 3 digits ie: theladder has 17 rungs. per page, define an arithmetic sequence i.e, the 25th term is 227. ‘a First 9 pages are numbered with single n prt, R- ‘with first term 192 (189 plus the 3 digits on page 100) and common difference 3, Letting 4, =1260: 192+(n-1)3=1260 189+3n=1260 n=357 So there are 99+357=456 pages in total. Exercise Bu Sutd=7 ...() (2)-2x(: d=6-14=-8 Substituting in (1): F(an+1)=1365 3n? += 2730 3n? +n—-2730=0 (3n+91)(n—30)=0 “.n=30 (asneZ') 14,=85, d u, <0 854(n—1)(-7)<0 ~7n+92<0 n>a134... 7 So the last positive term is 5. Si Flexas+(3-1)(-7)]=559 1, =6 =) 2(2u,+3d)=8 uj 43d=4 ... (2)-2x(1): d=4-12=-8 Substituting in (1): S,>10000, Fl2-8)+(n-1)x7] > 10000 S¢2m—19)> 10000 7h’ —19n—20000>0 n<~S2.1 or n>548_ (roots from GDC) So the smallest n such that S, >10000 is 55. © s,=3r°-2n flu +u,)=3n?—2n COMME! Remember that if you are given a formula for S,, then by substituting n= 1 you can immediately coleulate 5 =v, (©) There are 12 angles in the sequence, so n=12, The angle of the largest sector is twice the angle of the smallest sector, so u, = 2u,. Since the angles must add up to 360°, Sy =360 24 +,)=360 4, +2u, =60 3u, =60 4, =20 + smallest sector is 20°, wu _ 6 uy, 13 utdd 6 utlid 13 13(u, +4d)=6(u, +11d) Tu, =14d =u, =2d =) tus, =32 => u,(u,+2d)=32. ...(2) Substituting (1) into (2): 2d(2d+2d)=32 8d? =32 d=2 From (1): u, =2(42)=+44 Asall terms are positive, must have a= 4 and d=2. So Sooo 2 (209d) =50(2%44+99x2) 10300 Bipic 7 Goometis sscuerig. We need to find 112 + 140 + 154+ 182+ 196 +... + 980 +994 ‘This can be considered as the sum of two series: Sy =112+154+4196+...+994, series with. y =112, dy =42, my =22 S, =140+182+224+...+.980, series with a, =140, dy = 42, ny = 21 Sy 4Sy = 2llaxri2}s(22—nxa] + 2ifaxaso)+@1-yx42] =12166+11760 = 23926 Alternatively, the value can be calculated as the difference of two series: Sp =1124126+140+....+994, the 64 three-digit multiples of 14 Sq =126+168+...+.966, the 21 three- digit multiples of 42 (ie. multiples of both 21 and 14), which are excluded Sw-Sy =F [@xan2)+(61-x14] ~ 22x26) +(21-x42] = 35392-11466 = 23926 Exercise weur 62> u,r° e« eur ll = 7168 wy” = 7168 ie, 112r? = 7168 raed r=4 ut" 71684" = 1835008 Nlog0.4<-6 6 No~ > Tog(o4) N>15.08 ‘The least such N is 16, so the 16th term is the first to be less than 10°, u(P-r)= 224 a8 gat 3 (discounting the trivial case u, =0) ‘Using GDC to find the roots of the two possible cubics, there is one solution for each: .5 or 1.82 COMMENT In sequences and series, the leters a, d cand r usually have standard meanings, s0 take extra care in questions like this ‘one which use these letters in other ways, For the arithmetic progression: 4=0 4=1 satd=1 (1) w=b =at2d=b ... (2)-2x(): (3) LT Seqvences ond series 73 Soa=b Hence, substituting in (3): (a+2)(a— a=-2 or a= a=1=9b=1, which contradicts the requirement that a#b, sa=-2and b=4 uy, =u, +(32—1)d = 2+32x8 = 250 Sinceu,, t,,5 Msp (ie.10, 4,,, 250) form a ‘geometric sequence, 250=10r* P25 (Reject 10r =—50 since this clearly does not lie in the arithmetic sequence.) ye PAY RY Returning to the arithmetic sequence, ,+(m-l)d =2+(m=1)x8 @ a u,=3xs"" =3x5'xs"! =375x5"" Comparing this with the standard formula u, =u,r"", we find r=5. b From (a), 4, =375 (r-1) rod 395, =FT(0-1) - 1) 95 BO s-dirsr)=% Syaa(ltrertr)= 325 8 Dividing gives Lert tr 325 4 - 895 325_ 65 Ba Sater te? b Syatextx ex ext tat COMMENT bot In part (b} the result S, = S.,(1-°°) wa: “U(ltatx7+x+x'+2°) |p used, which enabled the answer to be read off immediately from part (0). Exercise Gq Ba s-15 =u(I+r)=15 ...0) S_=27 vee(2) Q+Q): (+r) 15. Each term of the series is positive, 2 rod 3 b From (2): R- Substituting (1) into (2): 32(1-r')=30 (r>0 asall terms are positive) S.-S, -2-at (3)) sali[u(d (This series has u, =2(4-3x), r= 4-3 2 Convergence occurs when -1 1.328 <4 (1-r")>1.328 l-r 08 0.6 1-0.4" >0.996 04" <0.004 nlog0.4 6.03 Require at least 7 terms for a sum greater than 1.328, Oy r=2t 2 Convergence occurs when -11328 <3 (© f(e)=142x+(2x)' +(2x)'+. geometric series with a= 1 and swith > since every term of the series is positive and it does not converge. Exercise Gg COMMENT Be careful to define the terms you use; in finance questions it wll ofen be critical whether you consider u, to be the value atthe start of year a or atthe end of yeor 1, If you are defining your own variables, colways state the definitions clearly at the start of the question Let u, represent the balance at the start of year n u, follows a geometric sequence with u, =1000, r= 1.03 a. 6th year interest =u, ~u, =1000(1.03"—1,03°) = 1000%1.03* x 0.03 =34.78 ‘The interest for the sixth year is £34.78, b The balance after six years is the balance at the start of the seventh year, 1, uy = 10001.03° = 1194.05, Balance after six years is £194.05. © Let u, be Lars’ salary in the nth year. 1, follows an arithmetic sequence with u, = 32000, d=1500 a thy =u, 419d =32000+19%1500 = 60500 In the twentieth year his salary will be $60500, , 21000000 5 (2m, +(n—1)4) 21000000 1n(52:500+1500n) > 2.000 000 15n? +625n— 2000020 3n? +125n-4000 20 Roots of this positive quadratic are 21.2 and ~20.5 (from GDC) on> 212 He will have earned more than $1 million after 22 years. Let u, be the balance at the start of year 1. 1, follows a geometric sequence with 1, = 5000, r= 1,063 After n full years the balance is the same as at the start of year n-+1: U,,, =5000%1.063" b Balance at the end of 5 years: 5000 1.063° = 6786.35 R- © { 5000x1.063" >10000 ii, 5000X1.063" > 10000 1.063" >2 ‘nlog1.063 > log2 n> eed Jogi.063 Balance will exceed $10000 after 12 full years. © Let, be the number of seats in row n. u, follows an arithmetic sequence with 4, =50, d=200 a $,28000 F (2m +(n—1)a)28000 ‘n(200n-100) 216000 2n? ~n-16020 Roots of this positive quadratic are 9.2 and -8.9 (from GDC) som>9.2 So 10 rows are required for there to be at least 8000 seats. b S 2 (axs0+9%200)=9500 S,==(2x50+4%200)= 2250 "The percentage of seats in the front half (rst 5 rows) is 22° = 23.79 9500 © 2 Balance at start of year n is 1001.05" +. V=100%1.05" = $265.33 b Balance at the end of month m is ral swos( 110% nas } > 2.6533, mnog( 1+ 28) > log2.6533 Topic 7H Mixed questions « m> 10g2.6533 = 234.6 It takes 235 months, equivalent to 19 years and 7 months. Let, be the number of miles run on day n. u, ollows an arithmetic sequence with =Ld=+ ‘ 4 a 8,226 F (2m +(n—1)a)226 a2) 252 44 Ww +7n-20820 Roots of this positive quadratic are 11,3 and~7.8 (from GDC) en>ll3 Afier 12 days the total distance exceeds 26 miles. 4, >26 4, +(n-1)d > 26 344526 4 n>4x26-3 n>101 On the 102nd day he runs more than 26 miles. G@ Leth, be the height the ball rises on the nth bounce, ie. ater hitting the ground times. 1, follows a geometric sequence with h, =2X08=16,r=08 a h,=16X08* =0.8192 metres tb ‘Total distance travelled at the end of bounce nis % 1.6(1-0.8") 8 =2+16(1-08") 2242 “The ball hits the ground for the 9th time at the end of bounce 8, 1,=2+16(1-0.8°)=15.3 metres COMMENT Note that the sum 7h, is doubled a becouse the ball goes up and down the same distance before it hits the ground ‘again. Let u, be the account balance at the beginning of year n, where 1 = 1 is 2010. 2 u,=1000 At the beginning of 2011, u, = 1000%1.04+1000 At the beginning of 2012, 14, = (1000 x1.04+1000)x1.04+1000 = 1000+1000%1.04+1000%1.04 ‘The pattern in (a) shows that, is the sum of a geometric sequence with = 25000(1.04"-1) © uw, 250000 25000(1.04" -1) > 50000 1.04" -122 10s" 23 rnlogl.042 log3 483-2801 Jogl.04 In the 29th year of saving, Samantha will have accumulated at least $5000. n> Mixed examination practice 7 Short questions Bu,=96 u,4+3d=96 (1) uy =15.6 uj +8d=156 ...(2) (2)-(): Substituting in (1): 6 5=2 (+4) 9 =3(6+156) =972 S,=2n?=n a §,=2xP-1=l>4,=1 $,=2x2?—-2: S,=2x3?-3 = Stu, my =5 $=, 4u, 914 =9 b Arithmetic sequence, with a=1,d=4 iu, =14(n-1)x4=4n-3 4.7 Sequences and series COMMENT As an alternative approach, recognise that ifS, is « quadratic with a zero constant term, then the context is an arithmetic sequence. State this and rewrite 5, in the form do) dn? 5,=B(20+d(0~)= ofo-g)-as compare coeficients to find d = 4 and ‘=|, and then use these values fo answer {o} and (b) directly, ©) Geometric sequence with u, ey nlog3>loglo® 6 m>gnies ‘The least such n is 13. u, =u, +4dandu, =u, +d Ms = 3uly u,+4d=3(u, +d) 2u,=d Arithmetic sequence {u, }hasu, =1. Geometric sequence {v, } has y, may, vl42d=r (0) w=, 14+3d=r7...(2) B<— 80 Mixed examination practice 7 Substituting (1) into (2): 1+3d=(1+2d)" 143d=144d+4d? 4d? +d=0 d(4d+1)=0 Sod=0 (corresponding tor=1and both{u, }and{v, } being the constant sequence 1, 1, 1,...) (corresponding tor ‘This is the sum of two infinite geometric series: 3] has sum to infinity 7 2 «the total value is $-+3=45 (@ his is an arithmetic series with u, =301 andd=7. ‘To find the number of terms: u, $600 301+(n-1)x7 $600 7n-7 $299 foal, from which it can be seen that the sequence is arithmetic, with 4, =3Ina~Finband a=—1inb. Ss = Bem +224) =2 (6ina-Inb-111n6) =69Ina~138Inb (e) Long questions (0) « Let A, be the amount in plan A after xn years; then {A, } is an arithmetic sequence with u, = 10800, d=800: A, =10000+800n Let B, be the amount in plan B after n years; then {B, } is a geometric sequence with u, = 10500, r=1.05: B, =10000x 1.05" © From GDG, intersection of the two graphs occurs at n=188, so for the first 19 years A, > By ie. plan Ais better than plan B. B Letu, be the number of bricks in row n, where row 1 is the top row. Then u,=1and ty, = arithmetic sequence with u, =1,d=2. f 8 u,=1+4(n-1)x2 =2n-1 b S,=36 Fl2m+(n-t)d) = 36 “(2+2n-2)=36 2 1 =36 n=6 S,=4u,+4 S(2H(n-1)x2)=4(2n—1)+4 a. There are n integers on the nth line b ‘TABLE 7ML.3 Line | Final integer 1 1 3 6 From the table it can be seen that the final integer on the nth line is the sum of the first n integers, ic. S, for an arithmetic sequence with u, = 1, d=1: Ss, = 24m) _n(nt1) ~ 2 Ltn “2 ) LT Sequences ond series 8) R , prety R* ‘The first integer on the nth line must be r ~ 1 less than the final integer: n(n+l) wtn—2n42_n?—nt2 2 2 ‘The integers on the nth line form an arithmetic sequence of n consecutive values from 2 2 =nt+2 7 POETS 10 12, so their sum is 2 2 2 2 (named, wen) n(In242) 02,1) 22 2 2 5 (nt +1)=16400 From GDC, n=32 ‘Consider the mortgage as held in one account (A) and the payments in a separate account (B).. ‘The mortgage account just rises at its interest rate: A, =15000%1.06". {A, } is a geometric sequence with A,=15000 x 1.06 andr=1.06. At the end of three years the mortgage account stands at A, = 150000 1.06° ‘The payments account works as B,,, =10000+1.06B,, since each year interest is added to the previous payments and then a new £10000 payment is made. ‘Therefore B, is a geometric series of m terms with a=10 000 and r= 1.06. At the end of three years, the payments account stands at 10 000(1+1.06+1.06") So, after three years, the balance is A,~ By 150000%1.06* ~(10000%1.06? +10000x1.06+10000) =150000x1.06° — 10000x1.06"—10000x1.06-10000 > Continuing the pattern: Balance after n years = A,B, (1.06"-1) 1.061 = 150000 1.06" 10000: (1.06"-1 =150000%1.06" 500000 —! ¢ Forthe balance at the end of n years to be $0, require (1.06"=1) 7S (.06"-1) 3 150000%1.06" —500 000: 150000%:1.06" < 500000: 0.91.06" $1.06"-1 1,06" 210 nz 395 1s logl.06 So the mortgage will be paid off after 40 years. ene 7 $3 Pinomial expansion Exercise GED ‘ ) General term of (x—2y)’ has the x 5 wwm(?) "(39 5 . Coefficient of this term is (her a (Hear =80 srs es teis( 5-09 b ()ear=-» =rs3 Termis Ge (-2y)' =-80x7y* (© General term of (3x+2y*)’has the form 5 efy.2)° (eo (2°) Require coefficient of x*y®, so r=3. 5 3 ‘Term is (Syosre') '=10x9x? x8y® = 720x298 Coefficient is 720 (© General term of (x*=3x"")’has the form Cer ery-(Jere Require 14—3r=5, so r=3 Term (Sar x =-945x° (©) General term of (2x~5x7)” has the form 12), yar (_e 2)" (Pex (-sx*J 12) unr, yr 12-3 (Pe (sy Require 12-3r=0, so r=4 ‘Term independent of x is ‘12 to (° )aesy x°=79200000 ©) General form of a term in the expansion of (1+3x)"is| ("los a(Mlos) = 42x 3n=42 n=14 {) General form of aterm inthe expansion of (1+2x)" is . \axy" | (slest =264x* nl) 5.4 = 264 2 n’-n-132=0 (n-12)(n411)=0 n=12 (reject negative solution =—11) (J) General form of a term in the expansion n of (1-5x)" is (Mess " 2(J}es0"=0sme nox tNer= 2), (-125)= 10500 n’—3n? +2n—504=0 n=9 (from GDC) {© General form of a term in the expansion of (+2x)"is (Mlorreer a (Jeresr =20412x7 n(n-1) 3" 2° 9 n(n=1)x3" =91854 n=7 (from GDC) =20412 Exercise (y+3y?)' =(y43y))f = 9°(143y)° General form of a term in the expansion of y*(143y)is (jer COMMENT If'a.common factor can be token outside the brackets, its offen simpler to do so before finding the general term. Birst four terms are: Arh (ord = y®(1+6(3y)+15(9y*)+20(27y*)) = y' +18y7 +135 y" +540)" B 0-H"a+x)=(a-x)0+0)"=(1-x*)" General form ofa term in the expansion _yig( gays of (1-x?) i(" \(-x?¥ COMMENT IF product can be simplified before expanding, this will generally lead to @ more rapid solution than expanding and then colculating the product, Always be alert for this kind of shortcut. First three terms are: (Bler({eon(Qir =1-10x7 +45" (1-2x+x*) =(a-x')" =0-x)” General form of a term in the expansion of (1-2) is (Pleas First four terms are: 20) yea? \eartal ane? \Ony? 0 \(—x)°+| 1 \(-x)'+] 2 \(-x)*+| 3 (-x) =1-20x+190x?-1140x" 8 Binomicl expansion WA) R* “P & General form of a term in the expansion of (1+x) is (\ expansion of (1+) is (14x) =143x43x7 42° General form of a term in the expansion 4 of (1+ mx)*is (omy v: expansion of (1+mx)* is (14mx)* =1+4mx-+6m*x? +. ‘Comparing coefficients of the product of these expansions with the given x2: 34 12m+6m? =93 mm? +2m=15=0 (m-3)(m+5)=0 sm=3, n=15 oF © General form of a term in the expansion 4 of (1+ kx) is (Shar «expansion of (1+kx)* is (L+kx)! = 144k +6K'x? +... General form ofa term in the expansion of (+x) ("le expansion of (1+) is (14x) 14m OD 2, . Topic 60 Binomial expansions as approxima mn Ro” ‘Comparing coefficients of the product of these expansions with the given 2 4ktn=13=9n=13-4k 6 abr Dar 6k? + 4k(13—4k)+(13-4k)(6—2k)=74 — 2k +2k+4=0 -Ln= a General form of a term in the 4 expansion of (-5x)* is (Fr t-se ‘The first 3 terms are (sJercssra lores +(S)eor ser =1x(3)!+4x(3)’(5x)+6x(3)' (Sx)? =81-540x+1350x" bh Require 3-5x =2.995, so x=0.001 > bla° = 81 = -540x' = -0.54 000001 = 1350x7 = 0.00135 Hence 2.995* = 81 - 0.54 + 0.00135 = 80.46135, comaE! In this type of question, find a value of x which mokes the first part of the question relevant to finding the approximation. In more complicated questions this may require some ingenuity. dp. @ Rounding to 6SF: the truncated term will tye negative, so this estimate should be rounded down: 2,995! = 80.4613 (6SF) COMMENT ‘Although a value like 80.46135 would ‘ormelly be rounded up, when using on expansion for approximation you should Consider the next term in determining svhether to round up or down if the value is oxaclly on the boundary. © © General form of aterm in the 7 expansion of (2+5x)’ is (Fler (Sx)! ‘The first 3 terms are (leven + (Heres + (Ceres = 128+7(64)(Sx)+21(32)(25x") =128+2240x+16 800: }y Require(2+5:)’ = 2.0057, so x= 0.001 el => 128x" = 128 L001 => 2240x' = 2,24 1.000001 = 16800x" = 0.0168 2,005" =128+ 2.24 +0.0168 = 130.2568 Rounding to 6SF: 2.005” = 130.257 Q: arse) -(JJers(JJaros +(Derext +. =128+1344x+6048x" +... bj Require 2+3x=2,3, 80 x=0.1. 128x° = 128 1344 60.48, 13443! 6048: Hence, approximately, 2.37 =128-+134.4 + 60.48 = 322.88 ii Require 2+3x = 2.03, sox =0.01 => 128x" = 128 13.44 x7 =0.0001 = 6048x7 = 0.6048 0.01 = 1344s! Hence, approximately, 2.037 = 128+13.44+0.6048 = 142.0448 by Approximation (ii) will be more accurate, on both an absolute and a relative basis, since the discarded terms (higher powers of x) reduce more rapidly in this case and are less significant to the total. Mixed examination practice 8 Short questions © General term of (2—x)” has the form 12), a(x) ; ‘Term in x° is 12) vo) as 3 5 (2) (-x)° =792x128x(-x' Coefficient is —10 1376 2-v8) (5 )ar (hora) “(Dera e(S\-2) = 3245(16)(~V2}+10(8)(2) +10(4)(~2V2}+5(2)(4)-4V2 = 232-1642 5 i ‘2 General form of a term in the expansion of (2+x)'is| (er x «expansion of (2+) is (jer x4 (Jer alt (er a+ (er e+ (Jer xt (Jars =32+5(16)x-+10(8)x?-+10(4)x? +5(2)x!-+2° 32+ 80x + 80x? +40x° +1024 +x° Require (2+x)' =2.01°, so x=0.01 32x? .000001, .00000001 3 General form of a term in the expansion of (1-2x)' is (Seer * expansion of (1- 2x)’ is (o}ces" +(}en + ( asi, = 143(-20)43( 4x2). =1-6x+12x7 +... General form of a term in the expansion of (3+4x)° is (Fler usr «expansion of (3+4x)' is (Jere (Jorans(Joraots.. = 243+5(81)(4x)+10(27)(16x") +... = 243+1620x+4320x" +... So the first 3 terms in the product are 243+(1620-6x 243) x +(4320-61620412%243)x” = 243+162x—2484x° 88 Mixed examinotion practice 8 D ‘Comparing coefficients of the product of these expansions with the given expression: rls x: 6+5m=n 2x7 15+30m+10m" = 415 mt +3m—40=0 (m-5)(m+8)=0 =x*—8x° +24x7-32x'+16x* sim=5, n=31 or m=-8,n=—34 (© General form of a term in the expansion Long questions ly. of(x+25) is © @ The graph of y=(x+2)' is the graph of . =x" after a translation (Jar (erery (eve ar. wy r or{ 3) Assen areas 2,0) ‘The term in x! corresponds to r=2 and (0,8). , y (© General form of a term in the expansion of (1+x) i(?}e Figure BML.1 Graph of y = (x+2)° ‘ae. » Gara(Jer2}are wnt Chow P+6x7 +12x+8 <. expansion of (1+ max)’ is (U-tme)’ =145mxe+10m?x? +... ) © Sinomial expension 89 my FR P/ ‘© Require that x+2= 2.001, so take x = =8 J x 12x! = 6x = 0.000006 Hence, 2,001’ = 8+0.012+0.000006 +0.000000001 = 8.012006001 94627 +12x416=0 (x+2)'+8=0 B tan(y-fO_ 0 g(x) Q+a) Vertical asymptote where denominator is zero: x =—2 ‘Axis intercepts at (0, h(0)) and where numerator is xra(0, 4) and (-1, 0) ‘5 b- General form of a term in the expansion of (1+) is (H\- +. expansion of f(x)is 1+5x+10x? +10x" +524 42° 4 7 General form of a term in the expansion of (2+)' i Jer x expansion of g(x)is (cle v-(HJer xt (Jere (jer 2 (Jere =16+32x+24x7 + 8x" +x" © i Using the expansions in (b), f(x) _ 8 45x48 +10x? +10x7 +541 (x) xt+8x°+ 24x? +32x416 48x44 24x? +3247 16x—(3x" +140? +2247 +111) - x1 48x" +24x7 +32x+16 4x? -72x7 48+(10x7 +50x? +85x+49] x4 +8x' +24x? +32x+16 =x-34 1x2 45087 485x449 x4 8x +247 + 32K +16 tek=3, a=10 pas ised extinct pectin. 3 1, pre ii, As x40, the rational function 10x? +50x? +85x+49 XUHBxT +24 +320+16 to zero, since the denominator has higher order than the numerator. therefore £0) x3 as x0 g(x) . Figure 8ML.2 Graph of y ae a B» say -[))+()}o1+(G)47 “(ke =143V2+3x2+2V2 =745y2 ‘> General form of a term in the expansion of (1+-¥2)" is ("ay « (i+av3)" Sova (tava) =$"|ensay So (1+x2)' +(1-x2)" Ear 9] ee] because the odd powers of x cancel while the even powers double up. ‘Taking x =1: (142) +(1-v2) Since the sum of integer values must be an integer, it follows that (1+N2)"+(1-V2)" is always an integer. COMMENT The above argument is more formal than striclly necessary, but you should be aware that talking about cancelling ‘terms! in a sum lacking powers of xis problematic because there is no obvious ordering for the terms. By introducing xand then evaluating at x=] as shown above, this problem can be completely between (1+~/2)" and the nearest whole number must in fact be (o-)|=p-v Require|1~J/2]' <10* <-nilog(J2-1)<-9 one log(V2=1 So the least such m is 24. 8,8inomial expansion 91 ne 7 —p. ©) 2 General form of a term in the For three consecutive terms to have the 1 same coefficient, require that a(r+2) nr n-r-l expansion of (a+x)' is zor The ratio of coefficients of the rth and ie alrti)en-r and a(r+2)en-r-1 =a(r+2) (r+1)th terms is But then ~(r+1 which is invalid since neN. This proves that there cannot be three consecutive terms with the same coefficient in a binomial expansion. COMMENT There are many ways to establish this from the equations set up in (b) and (c). There is no ‘best’ method in a question of this sort, so you should seek the quickest way to find a contradiction. b Ifa=1 and mis odd, say n=2k-+1, then arti B 2kt1-r ‘When r=k, the ratio of consecutive coefficients ‘This means that two consecutive terms have the same coefficient. c Replacing r with r+1 in the answer to (a): (r+2) (n-r-1) Circular measure and trigonometric functions Exercise GED Exercise GD cos(1+x)+cos( nx) © _y=psin(qx) has amplitude p, ie. y ranges = cosmcosx~sin msinx from -p to p. +cosmeosx+sin minx From the graph, p=5 =-cosx-0-cosx+0 ‘y= psin(qx) has period Tso the =—2cosx i 7 second positive zero oceurs at x= @ sine sin( 242i xt3t rom th pga ? 2 2 vom the graph, “E=m=2q= ‘sin(se+2n) (©) y=acos(x—6) has amplitude a, i. y =sinsetsinxcos +eosssin ranges from ~a to a. +sin.xcosm-+cos.xsin From the graph, a=2 an an y=acos(x—b) has a zero at x= 90° +b +i + cosxsin— +: SECO Feasxsiny Sine From the graph, the smallest positive =sinx+0+cosx—sinx +0+0—cosx+sinx solution is 110°, so b= 20° =sinx (©) © y=1+sin2x: amplitude 1; centre y= 1s period 7; axis intercepts comm! Note that sofa 2n) jinx by the (o.1) (F), (9) Periodicity ofthe sine function, so it isn’t .cos x: amplitude 2; centre necessary to expand the last term in the riod 2m; axis intercepts expression, io), (% °} ( ) y y=tsin2a y= eos we Figure 96.6 «9 Citevlar measure and trigonometric functions — 93, i > From Figure 9.6, there are two points of intersection in [0, 2], ~ two solutions. ‘The pattern of the two curves in Figure 9E.6 repeats every 2m. Since there are 2 solutions in an interval of length 2r, there must be 8 solutions in an interval of 8r. y= 2c0s(+x+60°): amplitude 2; centre y= 0; period m; axis intercepts (0,1), (30°, 0), (210°, 0) y At maximum point: cos(x+60°)=1 =0°, 360° .x=300° At minimum point: cos(x+60°)==1 = x+60°=180° 20° ‘The minimum and maximum points are (120°, -2) and (300°, 2) ‘The graph of y=2cos(x-+60°)-1 is the graph of y= 2cos(x+60°) after a translation > } so its minimum and maximum points are (120°, -3) and (300°, 1). Topic 9F, Modelling using trigonometric functions Exercise (1D y=acos(bt)-+m: amplitude a, perio« centre y=m From the graph: © o High tide will occur when dis a maximum, which occurs when sn( a 12, mtx 22 1-6 Low tide will occur when d is a ‘minimum, which occurs when Athigh tide, Atlow tide, d=16—7=9 metres Require so+7ein( Br) z19 ie. sn( Es)e2 2°72 From GDC: t ¢[1.69, 10.3] ‘This is equivalent to the period of time between 01:42 and 10:18. h=asin(kt): amplitude a, period = From the given information: amplitude=5=9a=5 ca riod = 10. peri =F lo>k== 5 a 3cm below the x-axis corresponds to h=3 sel) of) = = 3.79, 5.64 (3SF, from GDC) 2. 16.02, 898 5 ‘The point is 3m below the x-axis 6.02 seconds and 8,98 seconds after starting. =120—10cos400t: amplitude 10, centre h=120, period 2" =_™ 400 200 4 Greatest height is 120 —(-10) = 130m; least height is 120 — 10 = 110em > The time required to complete one full oscillation is the period, seconds 0157 © Greatest height occurs when 0s 4001 =—1 400t = 1 t= = 0.00785 (3SF) 400 i.e. 0.00785 seconds after release. Exercise (©) © For example: x=1 gives ™ arctanx = arcsinx = arceosx=0 arcsinx Clearly in this case arctan x #: arccosx COMMENT The false idea being disproved by counterexample here is that division “pastes through’ the inversion of a function — that for a function h(x) = st 4 it should follow that F(x) = generally not the case! Let 8 =arccosx, so x=cos6; then val e.aresinx = COMMENT This is easily verified using « compound angle identiy, as seen in Section 12B, © From (b),arcsinx-+arccosx = so the given equation becomes 2arctanx == 2 tan( Je 4 sinx+cos y= 0.6 -Q) cosx-siny=0.2 ...(2) From (1): c0s y= 0.6-sinx 1. y =arceos(0.6—sinx) From (2): sin y=cosx-0.2 sy =arcsin(cosx—0.2) _9 Grculor meggure gd trigonometric functions 95. b Need to solve arccos(0.6— sin x)= arcsin(cosx—0.2) From GDC, the solution is x= 0, and hence y= 0.927 (3SF) Mixed examination practice 9 Short questions © y=asind(x+c)+d has amplitude a and period/wavelength a a a=1.4, so amplitude is 1.4 metres b Distance between consecutive peaks metres (3SF) ey Figure 9MS.2 Groph of sin{2x}+2sin(6x) From the equation, itis clear that the period is at most x, which is 2m divided by the GCD of the coefficients of x in the trig functions. From the graph, the period is not less. than m, and so the period is exactly m. (©) y=acos(bt) has amplitude a and fod 2" read period © Period is ™ = 25n=78.5 seconds (3SF) 96 Mixed examination practice 9 pn Re ‘The amplitude is equivalent to the radius of the track, ie. 60 metres. e- Track length= 2x60 =120 =377 metres (3SF) © Speed = Distance b xef0,20]> x L(x)=0 asina(x-T) 0 Graph of y=3eina{ x) is maximum when n)_ ty; («2 )-Sesinia minimum when n)_x —7)2 22x, x =) £42k amplitude 3; y-intercept M1: (3,3) Ma: (If, 3) mi: (%,-3) m2: (1-3) Figure 9MS.4 Groph of y = sna x y=asin(bx) has maximum point at (5+) Ava 2b From the graph, the maximum is at (2, 5) oe Re2ssb=* and 2b 4 Long questions (Bo i y=sin(x—k)+¢ has a maximum at (E+her1) 2 By symmetry, point A is midway horizontally between the first two zeros, i.e. at x= 7 7 2n Beet 2 3 aka® 6 ‘The graph goes through the origin, 2 sin{o-E}ve=0 6 ~hec=0 2 1 Sc=> 2 So the coordinates of A are ( 3, 3 ay i) From @, kee 2 Period of y=sin(x—k)+-c is 2m, so the zeros are those shown in the question and the same at intervals of 2r. Within [-4n, 0), these are wan, —% 20,740 3 3 © i. For the equation (= as k <0, the first pair of solutions will be in the interval [%. anf and subsequent solutions will be at multiples of 2x further on, ie. in the inerals|*98 an, [!6n] and [#. sx} So there are only 8 solutions in [0, 9x]. COMMENT Its important to check thatthe nex! such interval is not needed too: in this case (=. tor] is wholly outside [0, 9x], so all the relevant intervals have been found. ii, Given that the smallest positive solution is of, the next solution, by E symmetry about x ad must be an (aS) = 3 3 The following solution, by periodicity, must be 27+ a So the next two solutions after o. are 107 wT oand 2n+a. 9 Girevlar measure and trigonometric functions 97 , o/ © iand ii ¢ Using GDCas necessary: so solutions of x-+tanx=n are intersections of the graphs in Figure 9ML.2. Given that x, is the first positive solution, by symmetry the other solutions in [0, 2] must be mand an=xy. ii Since each period of y= tanx extends infinitely in the positive and negative y directions, the line =x must intersect each period ofy=tana, so there are infinitely ‘many solutions. cor{ $e sin{ =) sina +sind sn( B)oorA-coo{ pina 2 2 =cosA-0 =c Mixed excmincionprocico® 1 BB .tanA= 3 or wvite a], tanA=3 > A= 1 tanA=— A v3 «the values are A=— or i ein ala ela = 6 a Minimum value of cos is -1, and the smallest positive value of x for which this occurs is x=. bi F(x) 027 x42) tanstion by = 6 o factor 2. and vertical stretch with scale r z ~ Dra Re {i Applying the two transformations in (i) to the minimum of cosx: the minimum point of y= of x4) is 6 (3, -1«2) ajie the 6 minimum value is ~2, and it occurs stem, 6 i Vertical asymptotes occur where the denominator is zero. ‘The minimum value of the denominator 2eos{x+ 2) 35 -2+3=1, so there are no vertical asymptotes. {i The maximum denominator value is 2+3=5, so the range of the denominator is (1, 5]. Hence the range of f(x) is [22 ]-ws, COMMENT Given the denominator is always strictly positive, the minimum of f(x) occurs when the denominator is at a maximum, and the maximum of F(x) occurs when the denominator is at a minimum, Trigonometric equations and identities Figure 10A.8 Solutions to sin x = + inom, rf sinx : has the 2 solutions But x, is outside the interval (1,1), so subtract 2: 2% —aq=—2 6 6 3 «the solutions are x=-7= 3-3sin?x—sinx= 3sin’ x+sinx—3=' nga TENTS sinx=0.847 or ~1.18 (reject as <-1) y 1.01 2.13 Figure 108.6 Solutions fo sinx = 0.847 in (0, 2m] ‘There are 2 solutions to sinx=0.847 in (0, an): x= aresin 0.847=1.01 x, =1-1.015 213 o 01, 2.13 @ & 2sin?x—3sinx=2 2sin? x—3sinx-2=0 (2sinx+1)(sinx—2)=0 or 2 (reject as >1) Figure 10B.8.1 Solutions to sinx = 0 in 1, m] sinx=0 has the 2 solutions 4 arein(-2) 30° and 2 =180°=x, = 210° But x, is outside the interval J0, 360°(, 80 add 360°: -30°+-360°= 330° ©. the solutions are x =210°, 330: Figure 10B.8.2 Solutions to cos | a in [-m, mI] sinxtanx = sin?x _ cosx=1=9x=0 sin? x cosx inta 106), bixed excminotiog practice, O. Ziyi oe my Re” ‘There are 2 solutions to each in the interval [0°, 360°]: cos0=0 =38=90°,270° 1 cos=-1 2 =30=120°, 240° :. the solutions are 90°, 120°, 240°, 270° (0) 6sin?x+cosx=4 6(1-cos'x)+cosx=4 6cos*x—cosx-2=0 (2cosx-+1)(3cosx—2)=0 12 cosx=-= or = 2 = 360° Figure 10MS.6 Solutions to cos x cos x= 2 in (0, 360°] ‘There are 2 solutions to each. For cosx = x =aeco(-4) 120° 2 x, = 360°—120° = 240° cose 3: For ; scuce2 et" x, 360°=48.2° = 312° = 48.2°,120°, 240°, 312° Gat An axe then xeon, 1]: ae ‘1 7 cos’ x+sin’ x sin’ x cos* x Let s= sin; then the equation becomes 163"(1-s*)=3 16s*-16s7+3=0 sinx=24 ox=2% 2 6 dine oxoat the solutions are +™, +7 6 3 Long questions x 219 95 1243n Figure 10ML.1 Graph ofy=18sn(2) between x= 0 ond the fist postive zero a The width of the river is the x-coordinate of the first positive zero: 3n=9.42 metres (35F) ‘The maximum width of the barge is the distance between the first two positive solutions of ussin(2) From GDC, the solutions are x=2.19 and x=7.24, so the maximum width of the barge is 7.24 — 2.19 = 5.05 metres. ‘The centre of the bridge is at 3n 2 = 471 metres. 2 This barge of width 2.5m, travelling along the centre of the river course, will be positioned in the interval [4.71-1.75, 4.71+1.75]=[2.96, 6.46] In this interval, y> 1 ssin{ 298) -150, so the maximum height of the barge is 1.50 metres above water level. Figure 10ML.2 Graph of lx cour + Jeo8 efor -2 s X <2 'b Since cos(a+2kn)=cosa for any integer ky O(s-+2n) =cos(x-+2n)+ 2 cos(2(0-42n)) =cos(x+2n)+ Leos(2x +4) 1 =cosx+e0s 2 =C(x) So C(a) is periodic with period a factor of 2n. From Figure 10ML.2 itis clear that the period is no less than 2n, so the period equals 2. From GDG, C(x) has maximum points at x=0, tn, +20 a cosx-h3cos2x'=0 1 cose if cos x-+c0sx— ey . = cos(—x) for all x * C(-x)= cos( =x) +c0s(-2) 1 =cosx+=cos2x 2 =C(x) ii Using parts (e)(i) and (b): © @ Repeated root = discriminant -16=0 b 4sin?@=5—kcosO 4(1-cos'@)=5-kcos@ cos" @—kcos0+1=0 © i f,()=4c0s*@—4c0s0 +1 From (a), 4x*-kx-+1=0 has a repeated root, so there is a single value of cos@ which satisfies £,(8)=0. i f,(0)=0 4cos’ @~-4cos0-+1=0 (2cose-1)' =0 cos= 2 Figure 10ML.3.1. Solutions to coso=} in 2m, 2n} ad <= Pray ‘There are 4 solutions. The first 2 are: =) iii Substituting x=1 into 4x” -kx+1=0: sok Figure TOML3.2 Soluions to cos@= kes ‘and cos@=1 in [-2n, 2n] From the graph, cos0'=+ has 4 solutions in [-2n, 2] 1 | 1 1 1 iv With k=5, (8) cos” @—-Scos8+1=0 (4c0s6~1){cos0-1)=0 and cos@=1 has 3 solutions, In total, there are 7 solutions in [-2n, 2r]. 10 Trigonometric equations and identities Geometry of triangles and circles To find AC, use the sine rule: COMMENT F , sinACB _ sinABC In questions on geometric shapes, if no diagram is given it is usually wise to AB Ac . draw a quick sketch, This reduces the ABsin ABC _ 6sin ABC ‘opportunity for errors of interpretation AC= "ACB sin35° cand makes it easier to check for the sense . wb. ee of an anewer. «'. two possible triangles exist: When checking for sense, always ‘ ©, 49,9°, 95.1° remember that in @tiongle, the widest ee ee ee een iia tal another with angles 35°, 130°, 14.9° and shortest side. AC=2.69cm. (©) By sine rule in triangle ABD: sin ABD _sinADB AD AB ADsin ADB. Sa) is (2) =arcsin| 6 3.6" = Ai=ars Figure 118.4 By the sine rule: By sine rule in triangle ABC: . . sin ACB _ sin ABC sinCAB 7 sinACB AB "AC Be ‘ Ap —aresin( ABSiNABC BCsinACB _ 8sin35° => ACB=aresin| AB 6 . > ‘Two solutions for CAB are = aresin( SSNS ) 8sin35° 8 x zaresin[ S50) =49.9° m1 2, = 180° —49.9° = 130.1° AC=180°—53.6°—37.1°=89.3° and hence ABC =180°~49.9°-35°=95.1° or 180°—130.1°-35°=14.9° (both solutions are viable). ‘110 Topic JBThesingye sinCAB= ray Re By sine rule in triangle ABC: AC sinBAC sin ABC _ ACsinBAC “sin ABC _85in89.3° © 8in53.6° =9.94.cm =9BC: By the sine rule: sinACB _ sinABC AB OAC ABsin ABC AG _12sin47° “8 =1.097 But since sinx <1 for any angle xin a triangle, this is not possible. Exercise s sin ACB= 2 kemy B Figure 116.4 BBS=130-15=115° By the cosine rule: (oP) +(PS)*-(BP)(PS)cosBEs 5° +2? ~2(5)(2)cos115° © By cosine rule in triangle ACD: bomen 2(AD\(CD) ren © +7710" ) 2(6)(7) =1003° +. BDC=180°-ADC=79.7° (AD) +(CD)'-(Acy } By sine rule in triangle BCD: BC__ bc sinBDC sinDBC DCsinBDC sinDBC _78in79.7° ~~ sin60° =7.95 1 XS795, =3BC= L\ Ag-3B Figure 116.6 By the cosine rule, . BC? = AB*+AC?—2(AB)(AC)cosBAC (643) = (4-3) 48° =2«8(4—3)%3 x 46x4+9= x? —6x+9464-8x424 20x = 88 x=44 4) Geometry of triangles and circles Figure 116.7 By the cosine rule, (LM) =(KLY +(KM)’ -2(KL)(KM)cosLKM pA in Figure 11D.4 Let PR=x. By the cosine rule, (RQ) =(PQ)'+(PR)'-2(PQ)(PR)cosRPQ 7 =8"+x7-2x8xc0s60° _ v2 f32-4x1x(-33) 49=64+x" 8x rr x°-8x+15=0 =2N22V41 (x-3)(x-5)=0 Since the length must be positive, x=3 or 5 Kan 22+ Jat Area of triangl Fea xsinnéa, . where x can take the two possible Exercise values above. Area difference = (PQ)sinRBQx(x,-*) Lge ¥3 =p x8x7x6-3) Figure 11D.3 =4N3 cm? By the cosine rule, LN= (LM) +(MNY—-2(LM)(MN)oosLNIN. Exercise yl2?-+7? —2(12)(7)cos135° 8 7.7 om, COMMENT ‘Area = 4(LM)(MN)sin LNIN There could be several possible answers 2 to this question, depending on which cuboid edge [if any] is included in ABC. IF the question had specified that no cuboid edges are included in ABC, then the solution narrows down to case (iv), which is the answer given in the back of the coursebook. - = x12x7 int 35° =29.7 cm? 112. Topic 11 Tigonomety in three dimensions . = DA Pe Case (i): If ABC includes a side of (For example, the angle between the sides length 12.5 (right triangle), then the oflengths 12.4 and 16.0 is sides are 12.5, 124?+1 14.5% Vi0?+7.3° =12.4 and — ( ~~ 302.4)(16.0) - }- 59.79.) f2.5*+10° +73" =17.6 Atea =} x12Ax14.5%609(72.7°) From trigonometry, the angles are 90°, =85.6cm" By trigonometry, the flagpole height BF is 12tan52°=15.4m Case (ii): If ABC includes a side of length 10 (right triangle), then the sides are 10, 125° +73 =14.5 and [12.5° +10°+7.3° =176 From trigonometry, the angles are 90°, arein( H4.6° and 55.4° 176 sre batostS=724 Case (ili): I ABC includes a side of length 7.3 (right triangle), then the sides are 7.3, vio +12.5" =16.0 and A 8 Figure 11£.4.1 N12.5* +10° +7.3° =17.6 ° ; . Let M be the midpoint of the base. If From trigonometry, the angles are 90°, one corner of the base is A and the apex } 245° and 65.5° of the pyramid is T, then by Pythagoras’ ed ‘Theorem, x7.3X16.0= 58.4 em? Cc Case (iv): If ABC includes no sides of the cuboid (oblique triangle), then the sides are V10°+7.3" =12.4, V0" +12.5° =16.0 and V12.5° +7. =145 By applying the cosine rule repeatedly, the angles are 47.6°, 59.7°, 72.7° A 8 Figure 116.4.2 11.Geomeiry of riongles and circles By the cosine rule, AB= (QB)? +(QA)?-2( QB)(QA)eos AQB. = (20.9 + 25° — 2(20.9)(25)cos75° =28.1m Q: tone and tanp= 2 RA=—*— and RB= tana By Pythagoras’ Theorem, (aB)’ =(Ray +(RB) ie, [=133em By Pythagoras’ Theorem, c= vi7?=12? = Vis =120em b- CMBis an isosceles right triangle with CMB=90° (the diagonals of a square meet ata right angle). <.CB= 2(CM)* =V2CM = v2Vi45 = 290 =17.0em Tepe VIE Tigonomelyin three mentions a ‘The angle subtended by the major arc is 2n-2.1,80 ©) The perimeter p is composed of three arcs with radius 5cm and angle wat =0.9375 radians . - peal se® b 0.9375 radians=0.9975 x80 * =x) =53.7° =Sm=15.7em ethene Mendel by ihemsiorG tetibe the length ofthe ar: then 1 o=— x07 r “1s 6 = p=25+8)+5.6 =3.75 =31.6cm #MCGN=24-3.75 (© The angle between the two 5cm sides is ‘=2.53 radians 0 =180°-2x15° x Figure 11.7 Length of major arc XY is 28m Figure 11R.11 {4 1Geomeiry triangles and circles 115 ‘The perimeter of the sector is made up of ‘two radii and an arc: ® On the cone, the slant height is 2 (8y f2-+[ >] =15, which isthe radius r of the sector. ‘The perimeter of the base is 18m, ‘which is the arc length / of the sector. oat ‘= 3.77 radians (or 216°) Exercise @ a-fre 2A 36a 10? =08 radians smaller angle= 2-337, which is equivalent to (2n—3.37)x 280° — 167° B NS Bibi 1G Areal sertoc| i Zs _ [x34 7V 12 = 90 =9.49.em 162° = 162x—2. = 2.827 radians 180 2A re 24 @ __ [exiB0 2.827 =13em +. shaded area =14.1-9=5.14 cm? ‘The perimeter of the sector is made up of two radii and an arc: p=2r+16 28=r(2+16) ar= 27780 36 Substituting (2) into (1): (2 +5) =? r 27 -77+6=0 (2r—3)(r-2)=0 r=15 or 2 So the radius is 1.5cm or 2em. (1) Let @be the minor sector angle. Then: P(2n-8) 2 re minor sector area A, = 2 AA, =15 ‘major sector area A, st >(2m-20)=15 (an-28) = 25 .0=n-0.6= 2.54 radians Exercise 2 ©) ® Minor segmentarea 5(0-sino) = 3 =(0-sind) =125(0-sin@)cm* b 12.5(@-sind)=15 6-sind=12 = 0=2.08 radians (from GDC) a By cosine rule in triangle PAQ: (PQ) =(AP)’ +(AQ)*-2(AP)(AQ)cos PAQ (20) =6 +6" -2x6x6% =36(2-V2) PQ=62-V2 b By cosine rule in triangle PBQ: PB)’ +(QB)" 2(PB)(QB) _#+4?~36(2-V2) ~ 244) 341 :PBQ=arccos0.341=70.1° cos PBQ: © Shaded area is the sum of two segments, one of radius 6 and angle radians and the other of radius 4 and angle 1.22 radians. suietae[aes( a8) afd x4? x(122- sn(1.22))] =1414+2.26 =3.67 cm? Mixed examination practice 11 Short questions GQ: COQ+T +E =n (angles ona straight line) x =c6a=% Total area =area COQ +area OABC-+area OAP = 2 x }eexrped(r 23 2 ao Ly 1b Triangle ABM is isosceles, with 12 BM= AB. = 28.9cm’ (3SF) ‘ i Using the cosine rule: c — = Perimeter =QC+CB+BA+AP+PO+0Q AM=\AB*+BM?—2(AB)(BM)cos ABM = 18.09 +8.09" 28,09" xcos48° =2x2474247%24742 3 6 =658m = 1 238em (BSE) Ge. Segment area =41°(0-sn8) p=2r+10 =+x7*(14-sin14) 36=2x10+100 2 pai =10.2em* (3SF) - a - 10° x1.6 en b By cosine rule in triangle OPQ, 2 2 PQ= /OP* +0Q?—2(OP)(OQ)cos® A = 7? +7? -2x7? xcos 14 Area. BB: Figure 11MS.3 Using the sine rule, sinABC _ sin ACB AC AB gc _ ACxsin ACB omhAR= BC= /(2v3)* +10" -2(10)(2/3)c05150° 7837.5" a - [evio0-ss{-“2) =0.854 ABC=aresin0.854 =H =58.7° or 180°-58.7°=121°(3SF) suite =2V43 By the cosine rule, BC? = AB* + AC?~2(AB)(AC)cos BAC 118 Mixed excmination practice 11 @ p=34 .2r+10=34 (1) Area =52 2Fr6=82 (2) a0 Substituting into (1): -17r+52=0 (r-13)(r-4)=0 r=13m or 4cm ( OF+A=90" because AT is a tangent. So, by Pythagoras’ Theorem, AT=V12?-6" = 108 = 63m +-Aeaof triangle OTA == x6V5 x6 Shaded area =area of OTA ~area of sector Liat =18V3-=x6 x— 3-56 XS =183-6n =123cm? a1 of B_ gi oO. ‘Area of segment BDCP = 5x2 ( sn’) =n-2 = 114m 'b ‘The semicircle with diameter BC has radius V2, so area of the region BECD is area of semicircle ~area of segment BDCP = 2B gy =n-(n-2) =2cm’ @. Angle of sector 2is 7-0 -.area of sector 2 b Total removed area is a semicircle with radius 2, os x12 _ x2? “remaining area =———. 2 2 =54-2n 17.7 cm? sinLKM _ sin42° 61 42 6.1 fast) By the sine rule, => LRM = aves or 180°~76.37° <- LMK = 180°—42°-LKM. =61.63° or 34,37° Since the triangle is obtuse, LKM =103.63° and LMK =34.37° = 7(LM)(KM)sin LN = $(6.y(4)sina4s7* =7.23. cm’ (3SF) Bo Cc = 10 Figure 11MS.12 11.Geometry of ranges ond circles a By the cosine rule, cos Abc - 4B" +BC*-AC* 2(AB)(BC) 8°+107-7" 2(8)(10) _us ~ 160 2 32, b sinABC=i—cos ABC = (2) 32. “5 /1024—529 1 == V495 32, 3 =i55 nv © Area =1(AB)(BC)sin ABC 1 3 =5Xx10x8x5 55 2 3 =P Vssem* (© a Shaded area is the difference between two sectors: 0 =5(100-100+20x-x*) _ 6x(20-x) ee b O=12: ‘Area =54.6 122120-9) 545 0.6x(20-x)=54.6 x7 20x+91=0 (x=13)(x-7)=0 o.8=7 (since x<10) 20 Mixed examination practice 114. prety Long questions QO: B LN A Figure 11ML.1 Using cosine rule in triangle AMB: ‘MB? = AM? +AB*~2(AM)(AB)cosMAB A #) a5¢-of (3) +5 af Zxscos0 x = 425-5xc0s0 x cost Using cosine rule in triangle ABC: BC? = AC? + AB*—2(AB)(BC)cosBAC. =x? +25—10xcos0 BC=MB => BC? = MB a ie, 27 +25-10xc0s8= 9" + 255x008 3x? 2X 50080 45x08 3x ” ce = a (asx #0) © x=5c089=/5-3 20 4 ne) 4 =41.4°(3SF) Figure 11ML.2 ASBT is a rhombus as each side has the same length r (the radius), Since ASB=90°, ASBT is a square, and s0SAT=90°, ABs the diagonal ofa square with side r. So, by Pythagoras Theorem, AB? =2r? AB=V2r Sector AST is a quarter circle. ne? s-Area AST =" 4 ‘The overlap consists of two sectors ‘minus the square ASBT. Area=2x2_7? 4 -(4) 2 ‘Area of minor segment =area of minor sector AOB ~area of triangle AOB b_ Area of major sector = area of circle area of minor sector * (@-sind) 2__—. A 2 3 Fen-0) 2(6-sind)=(2n-6) 26-2sinO=2n~-6 sind = 2 From GDC, @= 2.50 (3SE) Area=absinc (x+3)' =x? +(3x)’-2(x)(3x)cos0 x? 46x+9=x7 49x? —6x" cos 9x? —6x-9 6x" _3x?-2x-3 ~ 2x => c0s0= i cos*@=1-sin’@ (32) (2) =|=Sa—) ola 2x 3x ii Using GDC: x=1.24, 2.94 (3SF) Beacon #282 = 2x* =1.86, 0.172 11. Geometry ofriangles and circles Figure 11ML5 a By the cosine rule: . AB? =CA?+BC?-=2(CA)(BC)cos ACB 5? =x? +10°—2xx10c0s8=0 25-x*-100+20xcos6=0 x? = 20xcosO+75=0 Real solutions => discriminant A> 0: (-20cos6)’—4x1x7520 (20cos0)' = 300 20cos8<-10V3 or 20cos8>10V3 3 rseosos- or $F Se0s6S1 COMMENT Remember that -1< cos@ <1 always, so in the absence of other bounds these will sill hold. 150° $0< 180° wo) -s0 arccos(1)=0" 3 “2 ie, 0°<0<30? or 150°<0<180° cos $1=90°<8<30° fied exeminctinypgacticg A) a i AP*=(8-x)+(6-10)) 16x47 216x480 ii OP= yx? +100 By the cosine rule: OP? +AP?—OA?* 2(OP)(AP) (2? +100)+ (<4 cosOPA = _ 2x? -16x +80 2y(x? +100)(x*-16x+80) 2 8x+40 \(x? +100)(x*-16x-+80) © x=8=30BA=arccos( 40 [64x16 =38.7° (3SE) 4 OPA =60°= cosOBA=2 . x?-8x+40 _ © (x? +100)(x*-16x+80) From GDC, x=5.63 (3SF) © i If f(x)=cosOPA=1 then OPA=0, which happens when OAP isa straight line (so there is a solution). ii, When OAP forms a straight line, the gradients of OA and OP are equal: 6-0 10-0 @ ® § y=-16x? +160x—256 =~16(x?-10x+16) =-16{(x-3)’-25+16) =144-16(x-5)" ‘Maximum value of y occurs at x= 5 ii, Maximum value of y is 144 ii hax? +6*-2xxx6c08Z, =x" +36-12xcosZ iii cosZ= 2135" from (i) ax = 436-02) rom iy 12x _-64420x 12x _5x-16 3x xOxxxsinZ =3xsinZ = A*=9x" sin? Z AP =9x' sin?Z =9x"(1-cos*Z) af (5x16 =92x"| 1-| (bi ~( ( 3x y) by (b) Gi) =9x? — 25x" +160x—256 16x? +160x—256 i From (a)(ii), the maximum value of At is 144, so the maximum area is 12, ii From (a)(i), the maximum occurs when x= 5, for which z=10-5=5, Since x = z, the triangle is isosceles. By the same reasoning, O,BA: 7 and hence BAPO, is a rectangle, so PO, = AB (parallel sides in a rectangle have the same length). = PO, 25 _ love 35 o=aresin( VE 25 )- 1.369 (4SF) Length of chain =are AD+2AB+are BC =8(2n-20)+2x10V6 +3(20) = 85.6cm (3SF) mR” | | ) Further trigonometry * Sencha cos’ @+2cos?-1=0 1=tan?a, 2tan*@=1-1tan* a 1 8 = 20.955, 42.19 (1) 1-cos26=2sin’@ and 1+cos26=2cos’ 4, 1-cos20 _ 2sin’@ 14c0s20 2c05"@ =2(2cos*@-1)'-1 =tan’@ D & c0s40=2c0s*26-1 =8c0s' @—8cos’O-+1 © «Using tanza = 2222 i b c0s40=2c0s*20-1 1-tan’@ becomes 2tan’ a 1—tan* 2tan?@=6-6tan*o, Stan’ or=6 3 4 COMMENT In this case it is as fast to start again as p conver the answer rm (a ving cos? 9=1-sin?9. tan? a= v3 tana From (a): tan'(% B asin4x =2asin2xc0s2x, so asin 4x=b sin 2x 2a sin 2x cos 2x = b sin 2x Be os} omo(--5) » saxo 8}s(e® 3 3 (« (3) (5)) ={sinscos{®)+cosxsin( = 3 3 x 1 b (cept -(snsco(*}eosan(4)) oe} a _tan6-1 1+ tand tan( 0-7) =oan0 4 tan@-1 iFtand 6tan’ @+6tanO=tan6—1 tan’ O+5tand+1=0 (2tan@+1)(3tand+1)=0 stand tan@=—1 56 =-03224 n=282 3 oo ene) sinxcos| “ |+-cosxsin| = 4 4 =sn(x+") Maximum value is 1, and the smallest positive x value at which this occurs is x= rey pos 2cosxcos25°+ 2sin xsin25° = 2cos(x—25°) Maximum value is 2, and the smallest positive x value at which this occurs is x = 25°. sin(x-+0)=sinx By the compound angle formula, sin(x+0)=sinxcos0+cosxsind Assuming it is known that sin0=0, sinx=sinxcos0+0 sinx(cos0-1)=0 forall values of x 083A = cos(A+2A) ‘cosAcos2A —sin Asin2A =cosA(2cos? A~1)~sinA(2sin AcosA) = 2cos’ A-cosA~2cos Asin’ A =2cos' A—cosA~2c0s A(1~cos’ A) =4cos’ A-3cosA sin(A+2A) in Acos2A+cosAsin2A in A(1—2sin* A) +cos A(2sin Acos A) = sin A—2sin’ A+ 2sinAcos* A in A—2sin® A+2sin A(1—sin? A) sin A—4sin’ A =3sin A —3sin A(1—cos* A)~sin’ A =3sin Acos’ A-sin’ A Similarly, from (a): 083A =cos' A+3cosA(I~sin® A)-3cosA 208° A~3cosAsin® A sin3A ‘Therefore tan3A =———— cos3A 3sinAcos” A~sin’ A ~ cos’ A=3cos Asin’ A 126. Tic 128 Compo angle identies n pe Dividing through by cos’ A sin" and writing COMMENT You can derive this formula from a direct copproach through tan(3A) =tan(A +24), but this potentially generates a large cand unwieldy nested fraction; finding identities for sin3A and cos3A is more standard, and you will encounter this technique again in connection with De Moivre's Theorem in Chapter 15. sin(A+B)+sin(A-B) =(sinA cosB+cos Asin B) +(sin A cosB-cos Asin B) =2sin AcosB oy 2) a s—2) as 1" =2sinxcos= 6 a asinsco =) = 3cosx 3 nen 00st tanx= V8 v2 forxe[0, x] sin(A~B) cos(A-B) in AcosB—cos Asin B ~ cosAcosB+ sin AsinB Dividing through by cos AcosB gives tanA- 1+ tan AtanB tan(A-B)= tan(A~B)= < 5 Let@ be theanglof y=4x and, the angle of y=2x with the positive x-axis. 4 974 pyaae Figure 128.10 ‘Then 0=4,-9, Since the gradient of a line is equal to the tangent of the angle under the line, tang, =4, tang, =2 v.tan@= tan(¢,-¢,) 4-22 “14x29 D 2 0s(x+y)+cos(x-y) (cosxcos y—sinxsiny) +(cosxcos y+sin:xsin y) = 2cosxcosy b cos3x+cosx =cos(2x+x)+cos(2x-x) cos(2x-+x)+cos(2x—x)=3cos2x = 2cos2xcosx=3cos2x by (a) c0s2x(2cosx— =3c082x=0 or cosx +3 (no solutions) For x€[0, 2x} ie. 2x €[0, 4]: Let A=arccosx, so that cos = x; then co) L145) 2 2 Note that arccos has range [0, r], so — € [ 0, =} within this domain cosine is always non-negative, so there is no need to consider the negative ‘square root in this case. COMMENT ‘Always ensure that when you take & square root in a proof question, you either include both positive ond negative roots or explicitly reject one option, giving your reason. Without this, your of the foam a sinxa 2Ssinx+12cosx = 13sin(x+1.18) f(x) 913 f(x+1.18) ‘The transformations are a vertical stretch with scale factor 13 and a 118 horizontal translation ( } in either order. ° 3sinx—7cosx = Rsin(x—8) = Reos@sinx—RsinOcosx Reosd (3) = 0=arctan| ~ 3 ¢.3sinx—7cosx = V58 sin(x—1.17) Range of the function is[-V58, 58 ] 4cosx—Ssinx = Reos(x+a) tana= Reosa, =o arcian{>)=0396 -4cosx—Ssinx = V4 cos(x +0.896) 1Ocosx b 4cosx—Ssin cos(x +0.896 ‘The smallest positive solution occurs where x+0.896 =. = 0.675 V3sinx +cosx = Rcos(x-6) = ReosOcosx+ RsinOsinx R=(V3) +P 24 R=2 tang —Rsind _ V3. Reosd 1 = O=arctan( V3)=4 sy sinx+cos: =2eo4(x- 2) For x€[0, 2n} the maximum is at (£2) and the minimum is at ( s, 2) © sin2x+cos2x= Rsin(2x+a) = Reosasin2x + Rsinacos2x R=P4=25R=V2 _ Rsina_1 a= Roosa 1 = a=arctan(1): t jin 2x+cos2x = V2 sin (20+ sin2x+cos2x=1 Bein( 2+ sn(20+2) 4 ne[-m nJoanaxe Sel Primary solution for sin. , TR Periodic solutions 4, +2nn and A, + 2nm within the specified interval: Sm on (© sin’ 0+cot* @sin®@=sin*4(1+cot*6) =sin?of 14 2°28 sin’ sin? @(sin*@+cos?6) - sin’ =sin®@+c0s'6 sl |, xe[0, 2m] 08 Alternative algebraic solution, which ‘would be needed in a non-calculator ‘question specifying an exact solution: tanx+secx=4 sinx+1 =4 cosx sinx +1=4cosx 4cosx—sinx=1 Let 4cosx—sinx = Reos (x+c) = Reosccosx—Rsincsin x. Then Rast =17>R=V7 Rsine (4) = =c=arctan{ + Roose 4 4 Feo x+aictan(+)) wn) tance 08 (3SF) 12 Further tigonomelry 129 ‘The secondary solution se2n-arcon{ 1) -araan( 4) isnot a valid solution to the original problems from a right triangle with sides 1, 4 and 17 itis clear that 1 1)_™ othe -cos| retan{ 1 |=, so this ws (aa) (i) 2 would bea false solution x= 3%, for which both secx and tanx are undefined. COMMENT In the absence of other instructions, it is clearly more efficient to use the GDC than to write an algebraic solution at reat length Also, note that there is only a single solution {clear if you plot the function), but the olgebraic working suggests there should be two, with the other arising from the secondary solution to equation (*). However, os noted, this is @ false solution, which arises from cosx =0 since the rearrangement involved multiplying through by cosx, and this would need to be explicitly stated in an answer. Oo s)=tanetosee, ae[-2.2| From GDC: local maximuim is at (-0.715, -2.39), local minimum is at (0.715, 2.39) b Range of f(x) is 2, -2.39]U[2.39, of, tanxteotx= sinxcosx 1 sinxcosx i sinx cosx esexsecx Jppic 12 Reciprocal irigonometrical functions @ sno | 1-cosé sind _ 1+cos@——_(1-cos0)(1+.cos6) _2sin cos ~ 1=cos"6 BD @ sec?x—3tanx+1=0 But sec? x= e-l¢tan?x—3tanx+1=0 tan’ x—3tanx+2=0 b (tanx-1)(tanx-2)=0 = tanx=1 or 2 ¢ Forxe[0, an]: tanx=1=pxe2,2 a4 tanx=2=9x=1411, 4.25 m,), Se +. solutions are x 14.25 1 csc2x= 2sinxcosx Dividing through by sin’ x gives cot?x=1 2eotx ‘Then +=cosx (3) axceos{ 4 y and so f7(x)= arceo( +) ‘That is, the inverse function of secx is the arccosine of the reciprocal. Mixed examination practice 12 Short questions (0) 2tan?@-5secd-10=0 2(sec?@-1)-SsecO-10=0 2sec*@—SsecO—12=0 (2secO +3)(sec@—4)=0 wet=-2 ont For in the second quadrant, range of sec0 is |=, -1}, so only the first solution is valid. 3 s.sec0 ==> sec=—5 x 1 me 2 ens| x47 |=c0ss cosx—sing sinx 1 =Sc0sx: 2 ‘b Similar to (a), 1 B sinx =—cosx+~sinx 2 2 3sinx=0 for xe[-2n, 2m), x=-2n,—1, 0, 1, 2 & cos(8 +0)=cosOcos0—sinOsind =cos*O—sin"@ =cos@-(1-cos*6) =2cos*O-1 ‘aking x=2 in sin2x = 2sinxcoss: sind = 2sin%cos® 22 ‘Taking x Sin cosax+1=2e08x: 8 9 +1= 2cos"| 8) cotst-2e(2 tan6+cot@=3 Maltiplying through by tan@: tan? @—3tan+1=0 = tang= 2283-4 2 3tv5 2 8€[0°, 90°} = 20.9°, 69.1° COMMENT Nolice that the two answers must add up 10 90° because the original equation is symmetrical in tané and cot. VIS sin2x+ 5 cos2x = Rsin(2x+c2) = Reosasin2x +Rsinatcos2x R?=154+5=20=9 R= V20=2V5 _Rsina V5 _ 1 “Reo Vis V3 we-w jt 2.15 sin2x+V5 cos2x =2N5 n( 20-4 ) tana b f(s)e———2, fe s+-2NScin{ 2x i. Value of fis maximum when denominator is as small as possible: 2 _2(5+2V5) 5-25 5°-20 2 =5(5+2,5) 4 =2+2N5 3 4 pad as {i "The maximum occurs when sio(2+2)=-1 6 x last a sinfarcsine)=) defined |al>1 b Let x=cosys then y=arceosx andl sin? y=1—x? ssiny i.e. sin(arccosx)=v1—x? (Again, this is only valid over the natural domain of arceosx: (+1, 1].) © aresinx=arccosx: sin(aresinx)= sin (arceosx) 1 “2 Long questions © 2 We know that ABC=90° because AC=2r is the circle diameter and point B lies on the circumference. AB=2rsin0, BC=2rcos0 b Area of ABC=1(AB)(BC)=2r*sindcosd Using the double angle formula: Area of ABC =’ sin20 Triangle OBC is isosceles, so BOC=n~26 ‘Area of onc =5(0A\0C)sinBOC =F? sin(n=20) Since sinx=sin(n—x), Atea of OBC=1"sin(20) Lag 4 Area of OBC _ 3” 8in(20) ‘Area of ABC r°sin(26) 2 tanA+tanB 1-tanAtanB 2tanA tan(A +B); =>tan(A+A) tan135°= tan(-45°) Let tan67.5°=t; then 2 tanl35°= from (a) mn135°= >from (a) 2 24 rao Since 67.5° lies inthe first quadrant, t>0 142 ive. tan67.5°=1+V2 ‘y= acos px has amplitude a and «og 2m period P From the graph: y,(0)=12a=12 Halfa period is 1.5, so n 2n —s155 p= P pS yp=toee( 2) Amplitude is 0.9; period is 3 © y=12e04 #x}+09sn( 2) =Rsin x4) 3 R09? 41.2? =15? > R=15 Rsina 12 _4 r= Reosa 09 3 =a =arcan( 4) 0.927 (3SF) Ss y=lsi sin( 2£x--0927) Amplitude is 1.5; period is still 3. y= xso927=n Zon —0927)=1.06 (3SF) 13 x>0=9A>0927 Solving sinA= Primary solution A, aresin( $3) 105 Secondary solution A, = n— A, =2.09 Hence the first two positive solutions are x e(a-0927) = 0.0580, 0.557 (3SF) sm {3cos6- sind =rcos(8+c) =reoscrcos@—rsinasin® Pa3tl=4or=2 _ sine reosa 3 1 =azartan(-L)= ° (a) 6 tana. -seos8-sind=2co{ 047) Over a complete period the function 2eos{0-") has range [-2, 2] (4 12 Further trigonometry 133 a f(t}=P-3-3t+1 By inspection, f(-1)=0, so(t+1)isa factor of f(t) by the factor theorem. COMMENT Alternatively, plot the function on the GDC ond try to find o recognisable rational root. Hence f(t)=(t+1)("? +at+b) Expanding and comparing coefficients: 1°: b=1 is consistent with the value found above ef()=(e41)(0? —40-41) < mt Ro b tan(3A)=tan(A+2A) tanA+tan2A “ T=tanAtan2A tan A. diane, ietana ( 2tanA 1- toa) ‘Multiplying numerator and denominator by 1- tan’ A: tanA(1-tan’ A)+2tan A stan? A—2tan* A tan A—tan® A 1-3tan* A tan(3A)= tan45°=1 Because tan(xx-+180°)= tans, tan 405° = tan225°= tan 45°=1 Let t= tan15° or tan75° or tan135°; in any of these cases, the following is true, using the formula in (b): P37 —3t+1=0 (t+1(-4t+1)=0 by (@) yy ftva 4 2 1 or 1 or 243 Bach of these three values corresponds to one of tan15°, tan75° or tan135°. 01=>tan75°=24 V3 = pry) BS 1 3 vectors Oo B-w-HA 4) (3 =] -2|-| 1 The position vector of the midpoint is the mean of the position vectors of the end points: (3) (4 35 1 |+{-2] }=| -05 2 2) (5 1s © 0B=04+a5 =(2i-3/)+(i-J) =3i-4j > OD=OA+AD “)G)G) +. the coordinates of D are (10, ~2) M has position vector 3(p+4)=7(2i- j-ak+i+4j-k) 142 j-2k 2 ‘To form a parallelogram, CD = BA - OD=0C+@D i (© Require that a+ pb =k} 2 | for some k 3 2+3p) (3k ie] p |=) 2k 2+3p) (3k 2. p=2k from the second component. Substituting into 2+3p=3k: 246k=3k ake? 3 0 © Require that Ax+ y =| k | for some k 0 2A+4 0 ie.| 3241 J=|k At2 0 2A+4=0 from the first component shed 1 © Require that pa+b=4) 1 | for some k P¥2g) ie. | —p+1 3p+q ept2g=k Ml) ves(2) 3p+q=2k ...(3) )-Q): 2p+2g-1=0 (3)-2x(2): 5p+q-2=0 ‘Then 2x(5)—(4): 8p-3=0 3 =Pr5 and hence, substituting into (4): 1-2p_1 qeteet Exercise ce |/=12 =e Nate +c =12 vec? =12 6c? =144 =24 cata —2 AB=b-a=|-2 1 AB=V2?+2 +1 =3 nace? 2 (D M-05-oK atAb=| -A 242t 2a-1 —9-5t, Ja+ ab =5V2 (2a—2)' +A? +(2a-1)° =5V2 Require that AB=3 94? 122+! o(242t) +(440) +(945t) =9 3A? = 40-1 301? +1061+92=0 (34+5)(2-3)=0 15t°+53t+46=0 5 (t+2)(15t+23)=0 dao or 3 4 (1 a Require| k| -1 ||=6 for some k 1 ONG +P +7) = 6 18k? =36 katy 4 (PQ) =(e41)' + +(t-2)* Possible vectors are V2] ~1 =3F-2t45 1 2 b Require k] -1 |]=3 for some k>0 1 (same direction) eNO +P +L )K =3 6K =9 ke e = Erchoose positive root) val? s+ the vector is) 1 : 6410-3 ‘cos. 942541 J44449 1 ~wo 7] =87.7° (3SF) 38 Topic 13D Angles ne pry Re So the angle between AB and OA is 87,7° or 180° — 87,7° = 92.3°. earn cian) 2441 =arccos (mie ) ou - Vi7 53 = 40.0° (3SF) + ABCD isa parallelogram. b When k=1, 1 AB=| 0 |, AD=| 1 = areeos| ——— (ation =o =107°(3SF) ‘The angles of the parallelogram are 107° and 180°-9=73.2° For ABCD to be a rectangle, require AB-AD=0: k-2 o 4k-84+0+4k-2=0 8k=10 @ w-)1|.56 5. a AB-CA=-8-2+10=0 = BAC =90° =r Va41+49 16+ 44 -mo = 48.2° (3SF) ABC=180°—90°—48,2° = 41.8° =13.4(38F) ) Exercise a The vector equation can be written as the following system of equations: el) Substituting into (1): A+2x=-243(-3x-4) lix=-18 18 10 went andsoy= 7 b i (4-3)4(2+9)x=(-2-15)+(3-3)y, 1+llx=-17+0y 18, iL ii (12+2)+(6-6)x=(-6+10)+(942)y 14+0x=4411y ii Vectors were picked which were perpendicular to each of the direction vectors of the two lines, Ga COMME! For a twodimensional vector, find a perpendicular vector by exchanging the horizontal and vertical components and changing the sign of one of these. 1). . 2 3s perpendicular tol", -3 | 5 «| isperpendicularto| > (1), (2)_(0), (5 eu “3)P\s 1 Taking the scalar product with ( A} 34+0x= 64113 y= 2 3 ‘Taking the scalar product, wath : i 1B 2-1lx=15+0y => x= 7 in 2 a a-(b+c)=| -2 |, ype © 2 ab=0 and aa=t 6-24) (7 -A-17}, AC=! 4}, *.@-(2a—3b) = 2a-a~3a-l 5 2 b 0=45°= cos = a1 ABS| 2142 P-4=|p\|q|cos0 -3 1 sp W2= Dxlax 5 Kexe- aids -7(-6-24)+4(-2-17)+10=0 10A-16=0 0 =60° = cost q=16 With A= 1.6: -92 -7 BC=| -18.6|,AC=| 4 |, 5 2 22 AB=| 22.6 -3 BCA =90° from (a) 52: =a) (3a+b)-(a—3b)=0 = 68.7° 3a-a—9a-b+b-a—3b-b=0 BAC =180°-90°-68,7°=21.3° 3la|’ -3)6|°-8a-b=0 Since |a| =|b}, the first two terms cancel and this becomes =) SES 8a-b=0 2 .a-b=0 = 88.7 (3SF) which means that a and bare perpendicular. Area = Hocyac) +b BD=b-a b (a+b)-(b-a)=a-b-a-at+b-b-a-b =(bl'- © IFABCD isa thombus, then and hence (a+b)-(b-a) ‘This means that AC-BD = 0, so the diagonals are perpendicular. COMMENT Iris very important to know the defining qualities of various quadrilaterals: the square, rhombus, rectangle, poralelogtem, rapeziom and Kile. Ina {question ofthis sort, you con assume all other properties without proof, and only need to demonstrate the properly being requested. a OB=A0A, so OB and OA are parallel, hence B lies on (OA). 12-22 2-4 44h 0° => BC-BA=0 = (1-A)(2(12-24)+ (2-4) +4(4-42))=0 =9(1-A)(42-212)=0 “A= 2 (since A=1 is the degenerate case where OA = OB) 142. Topic 19F Areas c Figure 138.13 Since CBA = 90°, Bis the point on line (OA) that is closest to C. Hence the distance from C to the line (OA) is equal to BC: 8 A=2=BC=| 0 4 8 “.BC=|] 0 ||=/64+0+16 =4V5 @« d(,4,0) F(5, 0, 2) G(5,4,2) H(0, 4,2) lige aa b Area = |BEXBG) = Ve 45? +107 =V141=11.9 3SE) OC+CD. 7 4 =| 2}4] 0 |=) 2 3) (3) lo Exercise (EO So the coordinates of D are (11, 2,0). (ED) [2x4 =lal/b{sin30° — 1 =5x7xb b Area=|BAxBC 7K =175 Using xxx=0 and xxy=-yxx: (a-b)x(a+b) =axat+axb—bxa-bxb =0+axb+axb-0 =2axb (2a-3b)x(3a+2b) =6axa+4axb-9bxa-66xb =0+4axb+9axb—0 =13axb axb isa vector perpendicular to a, ‘Therefore (axb) -a is the scalar product of two perpendicular vectors, so it is zero, b (axb)-(a-b)=(axb) -a-(axb)-b =0-0 =0 @ 1 Gis the angle between vectors a and b, then by the properties of vector and scalar products: Jaxof+(a-b)? = (iall|sind) +((albjcos0)” =|al [of sin? 6+] cost P |b) (sin*@+cos*@) =a ei? Mixed examination practice 13 Short questions 2 = 1 Bilt ols -3) (aj}jl4 2\fa 5/}3 ajla 15412 1 o MD=MC+@ =13e+ 2 =15D-38 2 rn R” b Since ABCD is a rectangle, AB~ 2 10-1 9 5 |x} 1 f=] 144 [=] 5 |=91+5j+76 1 2 245 7, b Area =|ON x OL] =|nx1| NG 4547? = VIB = 124 (3SF) 3) (-1) ( -8 Ga |o}x| 5 |=|-1-3p 1} \p 5 15 3 5 1 b- Require that “| 4 \n some value k. Clearly, by inspecting the first and third components, k=-5, ‘Then, from the second component: n> = pr AY Rt (© Choosing H to be the origin, F AG as the positive x direction, EH as the positive y direction and DH as the positive z direction, we have oO 6 HA =| -4 |and HC=| 0 -3 3, “4 HA-HC AHIC=arecos| (i Al|HC| } wos =74.4° (3SF) (B \al=|)=Vsin® @+cos?@ =1 eran fall cos0 sind +sincos 1x1 =arecos(2cos0sin8) arcs ab= §((@10)+(a-0)) (a+8)-(a-2)) = §[le+4) (a+8)-(a+0)(0-») +(a-b)(a+b)-(a—b)-(a-b)] =F(e+2?-le-/}) {IN P @ Figure 13MS.8 b-a=MN, |a=PM, |=PN | +|b|'—24-b =|a +[6/' 2a cose ie, MN? = PM? +PN?—2(PM)(PN)cos@ Long questions 3) fa @e wali Fa 7 k 2 =| 0 k-7 b (AD) is perpendicular to (AB) AD-AB=0 2)\(2 0 | |=0 k-7) (4 —4-4(k-7)=0 4k=24 k=6 - = Pray © With k=5, 4) (7) (3 c=| 0 |+| 6 |=|6 a)\a)u i.e, the coordinates of C are (3, 6, 1) D. (Day(oe) «the coordinates of are( 3.3.2) o cos(ADC)= 4 c=, t+4 416 44 2 =4.06 (3SF) a The coordinates of P are (a, a°) b PO=-OP PS= @(-3+a")=0 a=0 or tv3 Since a>0, a= V3 1Sk-+15~18k=0 k=5 146 Mixed examinction procica IS 6) < Pray With a= 3, we know that OPS=90°, OP=Vi2 and PS=2 ssAen of PS = xD x2 = 248 @ » Ares ofbase= Naxs| b h=lelcos © Volume= height base area 1 1 = (close Jana) laxblle|cos® Since [AE] is perpendicular to the base, it is parallel to axb, and so the angle between axb and cis also 6. Volumes ZI(axb)-4 148 Topic 14B Solving problems with lines Lines and planes in space Exercise ant Equation ofline =| 1+¢ 4420 4 Att=-2, r=|—1|=0A Att=0, r=! SoA and B lie on I ALA, f=-2, and at B, #=0. Require AB=BG, so the point C must lie where t=2, «the coordinates of C are (0, 3, 0) 4 PQ=4q-p=|-2 3 7) (-4 sr=|i lea} 2) (3 AtB A=0, and at Q A=1. Require PR=3PQ, so the point R must lie where A=:3, since the distances are proportional to the differences in 2, Hence 7) (-4) (7) (-4 OR=|1 |+3-2| or | 1 |-3| 2 2) (3 2) \3 i.e, the coordinates of R are (~5, -5, 11) or (19, 7,~7) bt (© 2 2i-3j+6k gives a direction vector 2 of| -3 | for the line, 6 2 b | -3|-varov36=7 6 © As Pisa point on the line, 2 2 apelal -3 lal] -3]=214 6 6 AP=35>|4J=52=45 2) (2 2) (2 + OB=|1 |+5]-3] or |1 |-5]-3 4} lo 4) \o ice, the coordinates of P are (12, -14, 34) or (-8, 16, -26) Exercise COMMENT In these problems, assign unknowns to the values that need fo be determined, then use one or more standard equations which describe the geometry you are given. Solving the equations will give the values for the unknowns. 4424 © Since Clies on, OC=| 2-2 | for some? 1424 3-24 = p-c=| a 4-22, [PC] perpendicular to 1 = CB 6-4A-A+8-40=0 9A=14 act 9 + the coordinates of C are ( Let P be the point on the line that is closest to A(-1, 1, 2) 1-3t OP=| t |forsomet 2+t —243t PA=OA-OP=| 1-t |and 5-30 Gaarey 2 044A. Substituting into (1) gives 5-3A=4aA=-3 ‘Then, substituting into (3): 10+4A=-2=-9+741is valid, so the lines do intersect. Substituting, say, =I into the equation for ly gives (4, 1, ~2) as the coordinates of P. COMMENT Always use all three of the equations to tensure that the solution obtained from two of them is valid. In this case, the question states that the lines intersect, so this check serves fo reveal errors in working rather than determining whether or not the lines cre skew, but itis nonetheless worthwhile. 5 b When 1=2, r,=| 2 | 50. Q(5,2,5) 5 does lie on I. 5-34 5 -10-34 c Wis 1 =} 2 1 10+42 5+4A, for some value of 14Lines and planes in space Require that QM || 0 4 —10-34) (-3 a1 ‘| 0 |=0 5+4A 4 304+94+20+164=0 25a 10 =-2 the coordinates of M are (1, 1, 2) Figure 148.6 PMQ=90° Area of PMQ=2(PMV(MQ) =12.7(3SF) @: Atehous the hip sat ) b At¢hours, the second ship is at (2}le%s) 150. Topic 148 Solving probloms vit ines dtr. {us} =|18—94| When t=05, d=135 ‘The ships meet if there isa positive value of for which d=0: 18-91=0 =t=2 -+ the ships do meet, and this happens after 2 hours. d= 18km when [18~9¢|=18 9t-18=18 (since ¢>2) 9t=36 t=4 «the ships are 18km apart a further 2 hours after meeting, COMMENT Note that the other solution to the modulus equation [18-9 =18 is 18-9 =18 =1=0, which is already known since the ships stort 18km apart. a Attime f, the first aircraft is at 0) (3) (3 n=|5 [+e] -4 [=| 5-40 0 1 t 'b At time f, the second aircraft is at 0 5 St =| 0 |+e] 2 7 d=|,-n) = y(2t)' +(6t-5)'+(7- 24)" sod? = 40? +36t—60t+25+49—28t-+4t? = 440? -881+74 prety Re © Completing the square: a? =44(t~1)' +30230 ie. d£0 for all tand so the aircraft do not collide. From (c), the minimum d’ is 30, so the minimum distance dis 30 =5.48 km (3SF) Let P be the closest point to the origin on the line. 142A 242A | for some value of A 24a 2 Require that OP-| 2 |=0: 1 1424 ) (2 242 | 2 |= 24a 1 244A-4440424+4=0 A= h «the coordinates of P are (1, ~2 op=ViF4+4=3 a 2-t © « Atp]-1+54 |=|2+8 }s0 2434 1+3t Aq2-t (0) 1454 2) 24+3A=143t ...3) (+2): 6A-1=4 A= vr Substituting into (1): t=2-ae2 6 Substituting into (3): 2+3A5 2 = 1+3t is valid, so the lines do intersect. Substituting, say, A =3 into the equation for ), gives coordinates of P 1 -1 d,=|5), d,=| 1 3 3 Let @be the angle between the lines. ‘Then dd, @=arccos| (ci } =14549 = arceos| ( A+25+9 V1+1+9 -w a2 =48.5° (3SF) 1 ¢ Att=3, n=] 5 |=00,90Q 10 lies on =) (5) (at @ PO=| 5 |e) 19 JG) 1 10} “(27 33 WL =! 608 051 Figure 148.10 Let R be the closest point to Q on line /, ‘Then PQR is a triangle with PRQ=90, PQ= ea By trigonometry in PQR, QR=PQ sinQPR = 4.55 (3SE) and QPR = 48.5° 542A) (21) (-16+2a @® a PM=| 1-34 |-] 5 J=| 4-32 2+3a) (io) | -8+3a for some value of & 2 Require PM 16424 “| 4-34 |} 8430 3 -3244A+12494-24492=0 22A=44 4 «+. the coordinates of M are (9, -5, 8) 15 b When A=5, r=|-14 |=00, 7 80 Qlies on I. 152. Topic 148 Solving problemswithlines Figure 148.11 Require PQR to be an isosceles triangle, with base lying on I. It follows that M must lie at the midpoint of the base of the triangle, since (PM) is perpendicular to (QR). ‘Mhas A=2 and Qhas A=5, so R must have A= —1 for M to be the midpoint of (QRI. «. the coordinates of R are (3, 4,—1) 5 Atw=3, =| 2|=00, 6 80 Qles on, b By inspection, the two lines share a common position vector + P=(2,-1,0) COMME! Here you could carry out the standard procedure for finding the point of intersection, but it is much easier if you spot the common position vector! Figure 148.12 ip. prny R- Require PQR to be an isosceles triangle with PR=PQ (from Figure 148.12 there are two possible positions for R). 3 PQ=| 3 |=3V6 6 d |4,|=]| -2 ||=3> PR=3)A] ai 3} =3V6 A=4V6 «”. the coordinates of R are (2+V6, -1-2V6, 2V6) or (2-V6, -1+2V6, -2V6) Exercise a. The vector equation of the line is 1 3 r=| 4 |ra|—2 “1 3 Cartesian form: Pp Vector equation: Intersection with x-axis occurs where yaz=0; Require A such that -243A=0 fiaae 3 ‘This pair of simultaneous equations has consistent solution 4 =, so the line does intersect the x-axis, a(Zo, °) Let @be the angle between the 1 direction vectors a =| 0 | (for the 0 2 x-axis) andd=| 3 | (for the line). Then 2, coso = 4-2 lalla) Viz = 2 \e61.0° ona 5) 61.0° (3SF) © © Vector forms of the lines: 3 = e322 ye a4 Let Obe the angle between the 5 direction vectors d,=| 1 |and a 3 d,=| 0 | Then a1 = Ad 16 089 aa) S270 bares 5) 13.2° (3SF) ‘The lines intersect if there are A and that satisfy 3+5A=-1+3n ...{1) 2+A=1 --(2) pansy (3) Q)sasa Substituting into (1): ‘There is no consistent solution, so the two lines do not intersect. Parameterised form of the first line: (2+3v, -1+4v, -1+v) 5 Parameterised form of the second line: 1,=(5—H, -2-3u, 7+21) ‘The lines intersect if there are v and jt that satisfy the simultaneous equations 243V=5-H wl) <1+4v=-2-3u ...(2) -l+v=2u+7 — ...(3) (B)Sv=2n+8 Topic 14E Angles ‘ancl intersections between fi Substituting into (1): 2+6[+24=5— => 21=—7 Substituting into (2): -148U+32=-2-3=933 ‘These give the same value, = hence v=2. ‘Therefore the equations are consistent for U=-3, V=2 and the point of intersection is (8, 7, 1) By observation, at A=L, the line passes through (8, 7, Exercise a Normal vector for plane Il,: Angle @ between the planes is the same as the angle between the normal vectors: non, 13, oe ning) VINEE o=arccoe 25) =57° (to the nearest degree) (© Theline has vector equation 2) (3 r=|1|+a}2 o} 2 ‘The plane has equation r-| -1 |=5 sand planes For intersection, require 2) (3)\(2 1 }eaj2|}}=1 0) ila 2(2432)=(1422)-22=5 2 dal Substituting this value of A into the equation for the line: the intersection point is (5, 3, 1) COMME There are o great many ways to copproach vector problems, and a selection of methods for this question is given below. You should make sure you Understand all of them, and try to identity which approach you prefer in which circumstances. Each approach will have ils advantages, and if you can master mulliple approaches you can choose the most ecient for ony given problem ‘Method 1: using the parameterised form for one line and substituting it into the Cartesian form for the other line. For the first line, x=4-3A, y=143A, 2=2+A Substituting into the equation for the second line: 323A 3432 _ 3424 4 3 4 From the first and third expressions: 3-3A=34+20 9 A=0 But this is inconsistent with the second expression. ‘Therefore the two lines do not intersect. COMMENT Note thot ifthe lines do intersect, this method gives you the volue of the parameter 2. for the intersection point. ‘Method 2: using simultaneous equations for the parameterised forms ‘The second line has vector equation r=|~? J+] 3 2 An intersection of the two lines represents a solution to the simultaneous equations 4-3A=144u...(l) Ls3d=—2430 (2) 2h d=stmM ...(3) 3 3)=A=2u-— (3) HS Substituting into ( sou +4 = 10 But this is inconsistent for (2): 1404-2 +2 4 So there is no solution, COMMENT Note that ifthe lines do intersect, this method also gives you the value of the parameter A. (or jt) for the intersection point. 14tLines ond planes in space 155 Ay P —e Method 3: using the vector equations of the lines and taking scalar products with a ‘common normal. . 2 has vector equation r=| ln] 3 Pa 2 2 ‘The common normal to the two lines is 3 4 3 n=| 3 |x} 3|=| 10 1 2 21 For an intersection of the two lines: l+ul 3 ‘Taking the scalar product with fi: 3 4 2} 10 | feoa Il oa) la =a we [nl ln ‘This equation is invalid, therefore the two lines do not intersect. Although this method does not give the intersection point if the lines do intersect, inthe event that they do not, the diserepancy between the two sides of the invalid equation s the shortest distance Between the lines, which inthis case 72>. while this deta is not needed in question, method 3 would be extremely useful foro question which did tek forthe distance between the lines in subsequent par. Method 4: using projections ~ scalar products with two arbitrary vectors normal to one of the lines. ‘The second line i 2 has vector equation r=| “T+ 1) 3 = 2 2 For an intersection of the two lines: By inspection, | 0 |and| ~4 | are both -2, 0 perpendicular to the direction vector of the second line. ‘Taking the scalar product of (*) with 8-210 =1140p. wast 7 ‘These give inconsistent values of 2. ‘Therefore the lines do not intersect. This method, like the frst two, gives the intersection point if there is one. =V120" +30" +50" Geometrically it represents a projection ofthe second line onto the first in o ¥17800=135 (SSE) direction perpendicular to it. If wo such saath : projections are made, the same point will Oo. Substituting x=2, y=1, 2=6 into the be found ifthe lines intersect; otherwise equation for the plane: the projections will locate different =5(2)-3(1)-6=10-3-6=1= points, since Iwo skew lines will oppear LHS=5(2)-3(1) is to overlie each other at different places So the point P lies in the plane T. when viewed from different directions. ©) 12x-3y+52=60 a. Intersection with x-axis has y=z=0: 5 12x=60=9x=5 Normal to plane I is n=| ~3 .P(5,0,0) 4, ae a Angle @ between (PQ) and ITs the Tatersection with y-axis has «= 2=0: complement of the angle @ between -3y=60= y=-20 PO and n: .Q(0, -20, 0) cond= 35 B Intersection with z-axis has x= y=0: aa “Teis~ See COS e=12 +-sin(0)=sin(x—4)= cos(@) <-R(0, 0,12) -E- fe ui 45 © PQ=|PQ|= Vs?+2? +4? = /45 =3V5 ) 14 Lines and planes in space a er vy Let Re the point on TI such that (QR) is perpendicular to I ‘Then PRQ=90° and QRis the distance of Q from IT. = QR=PQsind = V45 x 3 1 1-2 -1)x| 2 |=|14+3 1 pi, ‘The normal vectors to the two planes 3 1 are n, =| —1|and n, =| 2 1 1 Since m1, =0, the normals are perpendicular and hence the planes are perpendicular. TT, is given by | 1) (3 1 |} -1 |=4#17, so M does not ajli lie in 11, 1 TH, is given by r-| 2 - 1)(1 1 |.| 2 |=1#17, 80 M does not a)la lie in TI. ‘The vector from (a) is perpendicular to the normal vectors of both planes, so it is the direction vector of the line of intersection, Se Hence the vector equation of line parallel to this direction and passing through M is given by a) fa 1|eal 4 2 1) (3) (-3+10) (7 3 |x| 5 |=] -6+1 |=|-s 2} (1) (5-9) (4 @. b i. Substituting x= y=z=0 into the equation of either plane leads to a ‘true statement 0= 0, so the origin lies in both planes, and hence their intersection also contains the origin. ii ‘The normal vectors for the planes are 1 3 3 |andn,=| 5 2 1 “The direction found in (a), which is perpendicular to both normal vectors, is the direction of the line of intersection. It was shown in (i) that the origin lies on the line of intersection, ‘Therefore the vector equation of the line of intersection is 7 r=A\-5 4 Substituting x=7A, y=-SA, 2=—44 into the equation for Tl: 7A-5(-5A)+(-44)=8 28A=8 azz 7 4 the point of intersection of the three 10 8 2-= pamis(2,-2,-2) - prt, Re Exercise () From GDC, the intersection point is _ Alternatively, using algebraic elimination: 3xt yt 228 0) -7xt3y+ z=2 (2) xt y+3z=0 (3) (1)-(3): 2x-22=8 oA) 3x(1)—(2): 16x+2z=22 ...(5) (4)+(5): Iex= 30-923 Then (4) = 2=x-4=~7 and (1) = y=8-2-34= 16 1 retin eins (816-2) + intersection points (5. 48,2 Using Gaussian elimination: x =2 fl) xty-z=7 (2) axtyt+z=3—...(3) @M o« Q-G@ oy (3)-2x(1) yt QM * ( (5)-(@) ())>z=-3 Then (4) => y=5+ ©. intersection point is (2, 2, -3) 0) (2)-2x(1) wn (4) (3) +B) Clearly these are consistent equations, but the system has only two independent equations, so the intersection is a line. Let x=tsthen z= 2t-land y=3-z=4-2t. «the line has equation oy fa r=] 4 [41-2 -1) \2 x4 2 © x-2ytz=5 1) Qxt yte=t —...(2) xt2y-z=-2 ...(3) (Cartesian form () x-2y+ 2=5 (2)-2«() ()-Q) Q) 4x(4) 5x(5) 20y-10z=-35. () (6) (7)-) ()=2=-2 -() Sy+7z=-32 ...(4) —Sy-72 «(5) 2x(2)-3x(1) 2x(3)-(1) Clearly, (4) and (5) are inconsistent, and so this system of equations has no solution, ie, there is no point at which the three planes all intersect, (a axt y-22=0 0) x-2y- z=2 ...(2) 3x+4y-3z=d ...(3) @ 2x+ y-2z=0 (1) 2x(2)-(1) sy =4 (4) 2x(3)-3x(1) Sy =2d_...(5) For this to be a consistent set of equations, require d=—2 ‘b With d=~2, (2) and (3) are consistent but the system has only two independent ‘equations, so the intersection is a line: In the form of a vector equation, this is 2 5 1 r=|_4|+alo 5] li 0 160 epic 14F Intersection of three planes @® xy =4 ..@) sa) x = (3) () oxy =4 (2) yrzsl ()-@)yrz=d-4 (Q) xy = (2) (2)-(4) (z= d-3 (2) yat-z2 de: (x=44 y= 2 2 2 ©) » each plane is written in the form r-fi,=k,, where fy is the unit normal to plane IT, then k, is the perpendicular distance from the origin to the plane. as jon points ( 2. +. intersection point is | —>,—=, In all three cases, this distance is zero, hence the origin lies in all three planes. ‘The origin therefore lies in the intersection of the planes. COMME! Coherently explaining something which seems clear can sometimes be tricky. It would be equally valid to show that {0, 0, 0) is consistent with the equation of each plane. xty =0 x-Ay-22=0 fessye z=0 Q) 0 xty (=) sy+2: 2x(3)-() Sy+2z= ‘As always, if you can avoid having fractions in your answer, it will appear tidior and you will lower your chances cof making arithmetical errors. In the elimination process, consider multiplying to make terms match instead of dividing {or multiplying by fractions). Equations (4) and (5) are consistent, but the system has only two independent ‘equations, so the intersection is a line. Let x=2t; then y=—2t and 2=5t, so the equation of the line is 25 or, in vector form, 2 r=t|-2 5 “The direction vector is d= 21—2)+5k Oe x-2y+ 2=7 Qx+ y~3: x+ yraz= Q) x-2y (2)-2x(1) 5y= (3)-@) 3y-(I+a)z=—4 (1) xa Dyt (2)+5 (5)-3x(6) Equation (7) is invalid if, this value the planes do not intersect. Taking only (1) and (6), the line of intersection of TI, and IT, can be obtained: Let z=f5 then (6)= y=z-1=t-1 (I)x=742y-z=5+t So the equation of the line is, in vector form, 5) ft r=|-1 felt o} of; in Cartesian form, x—5= y+1=z xo yo ze...) 2x43y—Te=at4 ...(2) xtDy+pe= @ xy (2)-2«(1) 5y- (3)-@) By+(p+1)z=a° 42 (0 x yo zd (4) Sy-Sz=a+8 5x(5)-3x(4) (5p+20)z=5a"—3a-14 ... Sz=a+8 ) 14 Lines and planes in space 161 p Equation (6) is invalid if p= -4, unless 5a’ —3a—14=(Sa+7)(a—2)=0, in which case equation (6) degenerates to.0=0 and there are only two independent equations in the system, so that the intersection of the planes is aline. p=-dand (5a+7)(a-2)=0 = p=-tanda=—4 or2 With p=—4and a=2, equation (4) becomes 5y—5z=10=> y-z=2 Let z=f;then y=24+z=24¢ and (1)=9x=y+2- «the equation of the line is 0) (2 r=|2|+/1 o) Ui or, in Cartesian form, Exercise COMMENT As suggested in the preamble fo this exercise, there are mony ways to ‘approach these problems. Each of the ‘worked solutions below is on example only, and should not be taken as the ‘best’ way. You should try a variety of methods and see which of them fee! most intuitive. Always remember that when asked to find an intersection, you can fake two equations simultaneously = what is true for the general point r in one equation can be applied to the expression of rin the other ~ to find. parameters. Don't be afraid to try novel approaches ‘A normal to the plane is n= -1 Line with direction n which passes through (-3, -3, 4) has equation +. +3 on in Cartesian form, “> = 27> 4g ‘The plane has equation r-m=11 ‘The intersection satisfies both this plane equation and the line equation 3) 2 -3|+al 2 4 -1 ‘Taking the scalar product of the line equation with n: 11=-16+9A, 9A=27 ‘This describes the intersection point Q, which therefore has coordinates (3, 3,1) © Shortest distance from point P to plane Twill be the distance PQ. Since Q corresponds to 2, PQ=3)n) =3V242 417 =o 1 Normal to T1,:#, =| -3 1 3 Normal to I: 2, =| -9|=3n, 3 ‘The normal vectors are parallel, so the planes must be parallel. S162 Brie 4s Sirtegis for sing problems with ines and planes nh Re b Substituting x= y=2=0 into the equation for T, leads to a true statement 0=0, so the point (0, 0,0) does lie in TT. 1 c r=al-3 1 COMMENT The question is leading you to establish the position of point P where the line in {c) intersects plane IT,, and then calculate the distance OP. However, there is @ faster way fo answer the question: the distance between two parallel planes r-m=k ond r-m= ky is ao this is a result you can quote cand use. ‘The distance d between r 6 1, =0 is equal to (> rom =Ois equal tof a Vi+3+ 0 L “1 0-2 Bal o jxl1 |e} o+3 2) (3) (-a-0 b i Atthe intersection, 7) fay (1) (0 -3}et}o f=[ a fesfa 2) (2) 26) (3 Taking the scalar product of 2 equation (*) with | 3 | gives “1 -25+0t=-25+0s ‘This is a valid statement, so the two lines do intersect. 0 ii, By observation, | 1 | is a vector 0 perpendicular to the direction vector of the first line, ‘Taking the scalar product of 0 ‘equation (*) with | 1 | gives 0 “3+0t=lts ssa-4 «+ the intersection is at (1,-3, 14) (Asa check, this lies on the first line for t= 6.) 2 Anormal to plane ITis n=] 3 and a the plane contains point (1, 1, 26) +. the equation of His 26 in Cartesian form, ~2x+3y—Z=—25 or 2x—3y+2=25 perpendicular to AB perpendicular to AG | 14 Lines and planes in space R- b From (a), AD is anormal 1 to the plane containing points A, B and C. So the equation of plane TI is given by 7)\(-1 r-n=a-n=|0)-| 5 Distance from point D to the plane is equal to the length AD, since AD is perpendicular to II. m1 ja0}=| 5 | Fase = v0 + the coordinates of D, are (8, -5, -1) In vector form, |, is given by 2 3 r=|-1]+a)-1 2 1 ‘The equation of I, may be rewritten in standardised Cartesian form as > So in vector form I, is given by 5 3 r=| 1 feu} 4, 1 ‘The two lines have the same direction vector and so are parallel. b For A=4, in|, the position is 2 r=|-1]+4 2 So A (14, -5, 6) does lie on h. © Point B lies on |, and so has position 5 3 vector} 1 |+s1| -1 |forsomevalue ofp. 4 1 9453p 6-H 10+ Require that AB. 1 27+ 9-6 +U-10+1=0 lp=43 «. the coordinates of B are (h-iea) nou a faginanced been Land equals [ABl. a. I has general point (1+32, -1+44, 3-34) Substituting into the Cartesian form of L: 1343, 2 13+34=8A-2= 40-62 15=5A=42-92 ‘The consistent solution is A=3 =4A-15 20-34 +. the point of intersection is (10, 11, -6) 3 Direction vector off isd, =| 4 3 2 Direction vector ofl, is dy =| 1 1 7 4,xd,=)-9 5 z avvector perpendicular to both. 7 lines is | -9 5 Equation of the plane with normal containing a point with position vector a is given by r-n=a-n: 7) (0\(7 u}}-9 6) (-5 =7x-9y-5z=1 Remember that there are many easy ‘checks! for this answer; for example, substituting in the other known points on the lines, (1, -1, 3) and (-12, 0, -17), provides a quick check, and ifthe values work then you can be confident in your Area ABC= > |ABXAG, +10" +2? =} ms =5.32 (38E) 3 Thas normal n=) -10 | and passes 2 through point A(8, 0, 4) So equation of Tis given by r-n=a-n: -3) (8)\(-3 r|-10 |=|0 || -10 2) a2 ~3x-10y+2z=-16 or 3x-+10y—22=16 4. 14.bines and planes in space Line through D(~7, -28, 11) with direction m has equation 7 -3 -28 |+2)-10 u 2 © Substituting the general point on this line, (-7-34, -28-10, 11+2), into the Cartesian equation of the plane gives 3(-7-3A)+10(-28-10A)—2(11+22)=16 So the intersection point E of the line with the plane is (2, 2, 5). Distance from point D to the plane is =3V113 =319 (3SF) Volume of the pyramid is 3 (Area ABC)xDE Votume=Le( 4s 3 jou COMMENT Where parameters and coordinates of tersections are ‘nice’ numbers (i.e. egers or simple rational valves), it can be useful to keep distances in surd form when combining, and then calculations need not be subject to rounding errors or extensive use of calculator memory. © 2 Vector equation for I: 1 6 r 1 feali 4 5 b ‘The normal vector of the plane must be perpendicular to each vector in the plane. ‘As A and Bare both points in the plane, AB is a vector in the plane and so AB is perpendicular to n. Line ! lies in the plane, so its direction vector d is a vector in the plane and therefore d is perpendicular to n. 4 ‘AB=| 2 |, A vector perpendicular to 3 both AB and dis 4 6 13 ABxd=| 2 [| 1|=|-38 3, 5 8 13 Hence n may equal | -38 -8 Thhas normal m and contains point A. Its equation is given by r-n=a-n: = 13x-38y-82=-83 6 a Anormal to the plane is) -2 1 6 line thas equation r= A] 2 ith nes and planes = ty Rt ‘This intersects the plane where 6(64)—2(-2A)+1(A)=16 So the foot of the perpendicular is at ( 96 32 16 ) p( 2° 41 “The shortest distance from the plane to the origin is the distance OP: A normal vector to a plane a ax+by+ce=d is| b |. ¢ 1 normal to Tl, ism=| 0 [and -1 a normal to TI, is#,=| 0 f; since n, =—n, , the normal vectors are parallel and therefore the planes are parallel, Line through origin that is perpendicular to the two planes: 1 r=A) 0 a © General point on the line is given by (4,0,-A). i. Substituting the general point into the equation for IT, gives 2=4>A=2 Point of intersection is P, (2, 0, -2) Substituting the general point into the equation for Il, gives -2A=83A=-4 Point of intersection is P,(~4, 0, 4) Distance between the two planes is PP, =V6' +0? +6? =6V2 COMMEN' This could also have been determined without using the results from (b), os described in the comment associated with question 2(d} of this exercise: The distance between two parallel planes ronak and rene ky is col so the i distance between II, ond Tl, is [4-kal ve need to rewrite the equation for II, as x-z=-B, 80 that both equations Use the same normal vector n. 642; remember that you would Mixed examination practice 14 Short questions 3 ©) Direction vector is| 1}, and the line 0 passes through point (6, 0, 1) 6 3 + equation of ine is r=] 0 |+A] 1 1 0. 1Atines nd planes in space 167 Q)2)-@\-)) (7 uxv=| (3)(2)-(1)(2) |=| 4 (1(-1)-(2)(2)) (-5, The normals to the two planes are u and v, so the direction vector of the intersection line is d= 4x, since this is perpendicular to both normals. Using d-u=d- d-w=d-(Aut pv) Ad-wt pd-v ‘That is, the direction of the intersection line is perpendicular to w = Au+ uv for all values of A and 4. (F) Rewrite the Cartesian equation of the first, line as 1: 2=° 3 zl Vector equation for | Ty ‘The plane containing both lines must have normal vector n perpendicular to both direction vectors. Choose m=) 3 ‘The plane contains point (0,3, 1) in line Equation ofa plane with normal m passing through a point with position vector a has equation r-n=a-n: = 7x42y-32=3 Check that I, does lie in this plane: Ifx=2, y=-L, 7x+2y-32=14-2- (2,1, 3)also lies in the plane 7x+2y—3z=0. This plane therefore does indeed contain both |, and k, COMMENT The question effectively asserts that there is a plane containing the two lines. This need not be the case, of course, since a pair of lines exist together in a plane only if they are poralel or intersect; two skew lines cannot lie in the same plane. It is good practice to check that the solution is volid, ether by finding the point of intersection of |, and I, or by checking that a point on |, actually does lie in the proposed plane, as shown.

Вам также может понравиться